新SAT官方阅读60篇literature - 图文 下载本文

????????????????????????????????????????????????????????

Read me

????????College Board????????CB??????????????4????SAT????????????????????????????????????????????????CB??????????????????????????????????Khan Academy????????????????????????????????????????????????2015??????????????????????????????????????essay????????????SAT????????????????????????????????????,??????????????????????????????????????????????????????????????????????????????????????????????????????????????????????????????????????????????????????????????????????????????????????3??????????????????????????????????SAT????????????????????diagnostic test???????????????????????????? ????????????????????????????????????????????????????????????????????????????????????????????????????????????

????????????????????????????????????????SAT??????????CB??????????????????????????????????SAT????????????????????????????????????????????????????????????????????????????????????????????????????????????????????????????????????????????????????±1??????????????????????????????????????????????????????????????????????????????????????????????????????????????????????????????????????????????????????????????????????????

??????????????????????????????????????????????????????????????????????????????????????????????????????AAA Studio ????????????????????????????????????????????????????????????????????????????

1.??????released ????????????coming soon??????????????2-3????????pending ????????????2????????????

2.????????????????????????3??????????(level2 level3 level4)??3.0??CB??????????????????OG??????????????????

4.1.1~1.4??????????????????????????????,1.5~1.7??????????????SAT????????????????1.8??AAA Studio ????????????????????????

5.2.1~2.3??????????????????????????????????2.5????????????????????????????????????????????????????????????????????????

6.3??AAA Studio ????????????????????????????SAT????????

7.4??AAA Studio ????????????????SAT??????????????????????????????

8.5????????????????????????????????????OG 4????????????????1??PSAT ????????????????????????????????????

9.??????????????????????????AAA Studio???????????????????????????????????????????????????????????????????????????????????? www.khanacademy.org??

????????????????????????????

????????????????????????????????????????????????????????

Parts&Items0 Sample1 Reading1.1 Science1.2 Literature1.3 Historylevel2(7 exercises)level2(5 exercises)level2(6 exercises)ContentOf?cial sample questionsStatereleasedlevel3(7 exercises)level4(7 exercises)level3(5 exercises)level4(4 exercises)level3(5 exercises)level4(3 exercises)level3(4 exercises)level4(3 exercises)releasedreleasedreleasedreleasedreleased1.4 Social science level2(4 exercises)1.5 Science supplement1.6 Social science supplement1.7 Literature supplement1.8 AAA approaches to reading2 Language2.1 Argument2.2 Informative2.3 Narrative2.4 Basics3 Essay4 Vocabulary5 Full testslevel2(4 exercises)level2(5 exercises)level2(4 exercises)Excerpted and Adapted from real SAT Tests before the reformationreleasedreleasedpendinglevel3(3 exercises)level4(4 exercises)level3(6 exercises)level4(6 exercises)level3(4 exercises)level4(4 exercises)releasedreleasedreleasedreleasedPendingcoming soonreleasedexercises that focus on single sentence grammar and usageAAA approach to the new SAT essayAAA glossary for the new SAT4 Of?cial OG practice tests plus 1 of?cial PSAT practice test and 1 mini test????????????????????????????

????????????????????????????????????????????????????????

??????????

??????????????????????????????????????????????????????????????????????????????????????????????????????????????????????????????????????????????????????????????????????????????????????????????????????????????????????????????????????????????????????????????????????????????????????????????????????????????????????????????????????????????????????????????????????????????????????????????????????????????????????????????????????????????????????????????????????

??????????TOEFL GRE SAT??????????????????????????????????????????????????????????????????????????????????????????????????????????????????????????????????????????????????????????????????????????????????SAT????????????????????????????????????SAT??????????????????????????????????????????????????????????????????????????????????

??????????????????????????????????????????????????????????????????????????????????????????????????SAT????????????????????????????????????????????????????????????????????????????????????????????????SAT????????????????????????????4??????????????????????????????????????????????????????????????????????????????????????????????????????????????????????????????????????????????????????OG?????????????????????????????????????????????????????????????????????????????????????????????? ????????SAT??????????????????????????????????????????????????????????????????????????????????????????????????????????????????????????????????????????????????????????????????????????????????????????????????????????????????????????????????????????????????????????????????????????1.8??2.4??3????????????

??????????????????????????????????????????????????????????????????????????????????????????????????????????????????????????????????????????????????????????????????????????????????????????????????????????????????????????????????????????????????????????????????????????????????????????????????????????????????????????????????????????????????????????????????????????????????????????????????????????????????????????????????????????????????????????????????????????mental effort??????????????????????????????????????????????????????????????????????????????????????mental effort????????????????????????????????????????????????????????????????????????????????????????????mental effort ???????????????????????????????? “patio” ???????????????????????????????? “????” ????????????????????????????????mental effort??????????????????????????????????????????????????????????????????????????????mental effort ??????????????????????????????????????????????????????????????????????????????????????????????????“????”???????????????? “patio” ????????????????????3??????????????????????????????????????????????

????????????????????????????

????????????????????????????????????????????????????????

??????????????????????????????????????????????????????????????????????mental effort????????????????????????????????????????????????????????????????????????????????????????????????????????mental effort ????????????????????????????????????????????????????????????????????????????????????????????????????????????????????????????????????????????????????????????????????????????????????????????????AAA Studio????????????

????????????????/??????????????????????????????????????????????????1??/????????1????0??OG??????????????????/??????????????????????????????????????????????????????????????1.5??/????????2????????????????????????????1.1~1.4?? level2??1.8??????2.4??????4????????????????????????????????????????????????????????????????????????????????????????????????????????????????????????????????????????????????????????????????????????????????????????????????????????????/??????????????????????????????????????????????????????1.5??/????????4~5????????????2.1~2.3??level2??level32.4????4Merriam Webster’s Vocabulary Builder????3??????????????????1.1~1.4?? level3??1.81.5~1.7????????????????????????????

????????????????????????????????????????????????????????

????????????????????????????????????????????????????????????????50%??????10%????SAT????????????????????????????????????????????????????????????????????????????????????????????????????????????????????????????????5????mini test??????Merriam Webster’s Vocabulary Builder????????????????????????????????????????????/??????????????????????????????????????????1??/????????6????????????????????????????1.1~1.4?? level4????????2.1~2.3??level4??????4Merriam Webster’s Vocabulary Builder????3??????????5????PSAT??OG??????????????????????????????????????????????????????????????????????????????????????????????????????????????????????????????????????????????????????????????????????????????????????????????????????www.khanacademy.org/humanities/us-history????????????????????????????????????????????????????????Older man and the sea(Hemingway) Catcher in the Rye(Salinger) Adventures of Huckleberry Finn(Twain) Harry Potter(Rowling) Pride and Prejudice(Austin)??????????????????????????????????????????????????????????????????????????????????????????????????????????????????????????????SAT??????????????????????????CB????????????????????????????????????????????????????????????????????????????????????????????????????????15%~20%??????????25%??????????????????????????????????????????????????????????????????????????????

???????????????? 2016??3??????????????????????????????

????????????????????????????????????????????????????????

Reading Practices for the Redesigned SAT 1.1 Science Level 2

Find more here at AAA studio

Proudly presented by Khan Academy Humbly brought to you by AAA Studio

????????????????????????????

Reading Practices for the Redesigned SAT 1.2 Literature Level 2

Find more here at AAA studio

Proudly presented by Khan Academy Humbly brought to you by AAA Studio

??????????????????????????????????????????

Exercise 155This passage is excerpted from Jane Austen, Northanger Abbey, originally published in 1803.Line5101520253035404550Mrs. Allen was so long in dressing that they did not enter the ballroom till late. As for Mr. Allen, he repaired directly to the card-room. With more care for the safety of her new gown than for the comfort of her protegee, Mrs. Allen made her way through the throng of men by the door, as swiftly as the necessary caution would allow; Catherine, however, kept close at her side, and linked her arm too ?rmly within her friend's to be torn asunder by any common effort of a struggling assembly. Still they moved on—something better was yet in view; and by a continued exertion of strength and ingenuity they found themselves at last in the passage behind the highest bench. It was a splendid sight, and she began, for the ?rst time that evening, to feel herself at a ball: she longed to dance, but she had not an acquaintance in the room.Catherine began to feel something of disappointment—she was tired of being continually pressed against by people, the generality of whose faces possessed nothing to interest, and with all of whom she was so wholly unacquainted that she could not relieve the irksomeness of imprisonment by the exchange of a syllable with any of her fellow captives. They saw nothing of Mr. Allen; and after looking about them in vain for a more eligible situation, were obliged to sit down at the end of a table, at which a large party were already placed, without having anything to do there, or anybody to speak to, except each other.Mrs. Allen congratulated herself, as soon as they were seated, on having preserved her gown from injury. \have been very shocking to have it torn,\not it? It is such a delicate muslin. For my part I have not seen anything I like so well in the whole room, I assure you.\\have a single acquaintance here!\\is very uncomfortable indeed.\\look as if they wondered why we came here—we seem forcing ourselves into their party.\\large acquaintance here.\\\them directly. The Skinners were here last year—I wish they were here now.\\for us, you see.\\think we had better sit still, for one gets so tumbled in such a crowd! How is my head, my dear? Somebody gave me a push that has hurt it, I am afraid.\\you sure there is nobody you know in all this multitude of people? I think you must know somebody.\60657075\acquaintance here with all my heart, and then I should get you a partner. I should be so glad to have you dance. There goes a strange-looking woman! What an odd gown she has got on! How old-fashioned it is! Look at the back.\After some time they received an offer of tea from one of their neighbours; it was thankfully accepted, and this introduced a light conversation with the gentleman who offered it, which was the only time that anybody spoke to them during the evening, till they were discovered and joined by Mr. Allen when the dance was over.\had an agreeable ball.\\hide a great yawn.\we could have got a partner for her. I have been saying how glad I should be if the Skinners were here this winter instead of last; or if the Parrys had come, as they talked of once, she might have danced with George Parry. I am so sorry she has not had a partner!\\consolation.QUESTION 1 OF 11The main purpose of the ?rst paragraph is to???????????????? A establish the distant relationship between Mr. and Mrs. Allen. B show how claustrophobic and crowded the ballroom is. C describe the main characters' belated arrival at the ball. D introduce the friendship between Catherine and Mrs. Allen. QUESTION 2 OF 11The narrator implies that Catherine???????????????? A did not have an enjoyable time at the ball. B would rather not have to dance with anyone. C had a ?ne time at the ball in spite of herself. D does not like keeping the company of Mrs. Allen. QUESTION 3 OF 11Which choice provides the best evidence for the answer to the previous question????? A lines 3–9 (“With . . . assembly”)(“With more care for the safety of her new gown than for the comfort of her protegee, Mrs. Allen made her way through the throng of men by the door, as swiftly as the necessary caution would allow; Catherine, however, kept close at her side, and linked her arm too ?rmly within her friend's to be torn asunder by any common effort of a struggling assembly.”) B lines 12–14 (“It . . . room”)(“It was a splendid sight, and she began, for the ?rst time that evening, to feel herself at a ball: she longed to dance, but she had not an acquaintance in the room.”) ????????????????????????????????

??????????????????????????????????????????

??

??

C lines 15–20 (“Catherine . . . captives”)(“Catherine began to feel something of disappointment—she was tired of being continually pressed against by people, the generality of whose faces possessed nothing to interest, and with all of whom she was so wholly unacquainted that she could not relieve the irksomeness of imprisonment by the exchange of a syllable with any of her fellow captives.”) D lines 20–25 (“They . . . other”)(“They saw nothing of Mr. Allen; and after looking about them in vain for a more eligible situation, were obliged to sit down at the end of a table, at which a large party were already placed, without having anything to do there, or anybody to speak to, except each other.”) ??

??

A lines 31–34 (“How . . . indeed”)(“\is,\acquaintance here!\??

??

B lines 35–39 (“What . . . here”)(“\gentlemen and ladies at this table look as if they wondered why we came here—we seem forcing ourselves into their party.\ C lines 40–43 (“I . . . now”)(“\somebody to go to.\ D lines 50–52 (“But . . . somebody”)(“But, dear Mrs. Allen, are you sure there is nobody you know in all this multitude of people? I think you must know somebody.\????

????

????

QUESTION 4 OF 11

In line 11 (“ingenuity”), “ingenuity” most nearly means????????

????????

A brilliance. B dexterity. C inventiveness. D intellect.

QUESTION 9 OF 11

As used in line 65 (“agreeable”) , “agreeable” most nearly means????????

????????

A enjoyable. B acceptable. C tolerable. D common.

QUESTION 5 OF 11

The description of Mrs. Allen in lines 26–30 (“Mrs. Allen congratulated herself, as soon as they were seated, on having preserved her gown from injury. \\anything I like so well in the whole room, I assure you.””) primarily serves to????????

????????

A illustrate her unruf?ed personality. B highlight her scrupulousness. C show her expensive tastes. D establish her sense of vanity.

QUESTION 10 OF 11

Mrs. Allen’s response in lines 53–55 (“I . . . dance”)(“\word—I wish I did. I wish I had a large acquaintance here with all my heart, and then I should get you a partner. I should be so glad to have you dance.”)mainly serves to????????

????????

A show how shunned they are in the midst of the ball. B highlight her attempts to expand her social circle. C emphasize the goal of ?nding Catherine a partner. D demonstrate their inability to mingle with new acquaintances.

QUESTION 6 OF 11

According to the passage, Catherine is “uncomfortable” because????????

????????

A it is too noisy, crowded, and warm. B they do not know anyone at the ball. C they have arrived at the ball very late. D their dresses are at risk of being torn.

QUESTION 11 OF 11

The conversation between Mr. Allen and Catherine in lines 64–67 (“Well . . . yawn”) (“\had an agreeable ball.\????????

????????

A only being polite to Mr. Allen to conceal her real feelings. B incredibly tired and therefore not paying attention to Mr. Allen. C extremely bored and doesn’t care if Mr. Allen notices it. D offended by Mr. Allen’s question and answers sarcastically.

QUESTION 7 OF 11

Which statement best characterizes the relationship between Mrs. Allen and Catherine?????????

????????

A Catherine is irritated by Mrs. Allen but defers to her in conversation B Catherine is amused by Mrs. Allen but is frustrated by her lack of social connections C Catherine is impressed by Mrs. Allen and is interested in her social contacts D Catherine is angered by Mrs. Allen and believes her social tactics are ineffective

QUESTION 8 OF 11

Which choice provides the best evidence for the answer to the previous question?

????????????????????????????

??????????????????????????????????????????

Exercise 255This passage is excerpted from L.M. Montgomery, “The Gossip of Valley View,” originally published in 1910.Line5101520253035404550It was the ?rst of April, and Julius Barrett, aged fourteen, perched on his father's gatepost, watched ruefully the low descending sun, and counted that day lost. He had not succeeded in \repeatedly. One and all, old and young, of his intended victims had been too wary for Julius. Hence, Julius was disgusted and ready for anything in the way of a stratagem or a spoil.The Barrett gatepost topped the highest hill in Valley View. Julius could see the entire settlement, from \Everett's farm, a mile to the west, to Adelia Williams's weather-grey little house on a moonrise slope to the east. He was gazing moodily down the muddy road when Dan Chester, homeward bound from the post of?ce, came riding sloppily along on his grey mare and pulled up by the Barrett gate to hand a paper to Julius. Dan was a young man who took life and himself very seriously. He seldom smiled, never joked, and had a Washingtonian reputation for veracity. Dan had never told a conscious falsehood in his life; he never even exaggerated.Julius, beholding Dan's solemn face, was seized with a perfectly irresistible desire to \moment his eye caught the dazzling re?ection of the setting sun on the windows of Adelia Williams's house, and he had an inspiration little short of diabolical. \news, Dan?\\\kind of a family affair, but then Adelia didn't say not to, and anyway it'll be all over the place soon. So I'll tell you, Dan, if you'll promise never to tell who told you. Adelia Williams and Young Thomas Everett are going to be married.\Julius delivered himself of this tremendous lie with a transparently earnest countenance. Yet Dan, credulous as he was, could not believe it all at once.\\last night and told Ma all about it. Ma's her cousin, you know. The wedding is to be in June, and Adelia asked Ma to help her get her quilts and things ready.\Julius reeled all this off so glibly that Dan ?nally believed the story, despite the fact that the people thus coupled together in prospective matrimony were the very last people in Valley View who could have been expected to marry each other. Young Thomas was a con?rmed bachelor of ?fty, and Adelia Williams was forty; they were not supposed to be even well acquainted, as the Everetts and the Williamses had never been very friendly, although no open feud existed between them.Nevertheless, in view of Julius's circumstantial statements, the amazing news must be true, and Dan was instantly agog to carry it further. Julius watched Dan and the grey mare out 6065707580of sight, fairly writhing with ecstasy. Oh, but Dan had been easy! The story would be all over Valley View in twenty-four hours. Julius laughed until he came near to falling off the gatepost.At this point Julius and Danny drop out of our story, and Young Thomas enters.It was two days later when Young Thomas heard that he was to be married to Adelia Williams in June. Eben Clark, the blacksmith, told him when he went to the forge to get his horse shod. Young Thomas laughed his big jolly laugh. Valley View gossip had been marrying him off for the last thirty years, although never before to Adelia Williams.\Eben grinned broadly. \Tom,\was glad to hear it, although I was mighty surprised. I never thought of you and Adelia. But she's a ?ne little woman and will make you a capital wife.\Young Thomas grunted and drove away. He had a good deal of business to do that day, involving calls at various places—the store for molasses, the mill for ?our, Jim Bentley's for seed grain, the doctor's for toothache drops for his housekeeper, the post of?ce for mail—and at each and every place he was joked about his approaching marriage. In the end it rather annoyed Young Thomas. He drove home at last in what was for him something of a temper. How on earth had that fool story started? With such detailed circumstantiality of rugs and quilts, too? Adelia Williams must be going to marry somebody, and the Valley View gossips, unable to locate the man, had guessed Young Thomas.QUESTION 1 OF 11Over the course of the passage, the main focus shifts from???????????????? A a character’s attempt at a practical joke to the effect felt by another character. B a depiction of a small town to the relationships among the characters in the town. C a character’s humorous behavior to the evolution of a story passed around the town. D a description of a friendship to a discussion of a prank perpetuated by the town. QUESTION 2 OF 11Which statement best characterizes the relationship between Julius and Dan????????????????? A Dan is trusting of Julius and does not question his story about Thomas. B Dan is skeptical by nature but trusts Julius to always tell him the truth. C Dan's disposition makes him an attractive target for Julius' plan. D Dan’s attitude towards Julius illustrates an imbalanced friendship. ????????????????????????????

??????????????????????????????????????????

QUESTION 3 OF 11

As used in line 2 (“ruefully”) , the phrase “ruefully” most nearly means????????

????????

A mournfully. B apologetically. C repentantly. D angrily.

??

??

A lines 16–20 (“Dan . . . exaggerated”)(“Dan was a young man who took life and himself very seriously. He seldom smiled, never joked, and had a Washingtonian reputation for veracity. Dan had never told a conscious falsehood in his life; he never even exaggerated.”) ??

??

B lines 21–22 (“Julius . . . him”)(“Julius, beholding Dan's solemn face, was seized with a perfectly irresistible desire to \ C lines 34–35 (“Yet . . . once”)(“Yet Dan, credulous as he was, could not believe it all at once.”) D lines 53–54 (“Oh . . . easy”)(“Oh, but Dan had been easy!”) QUESTION 4 OF 11

As used in line 18 (“veracity.”), “veracity” most nearly means????????

????????

A candor. B truthfulness. C verisimilitude. D impartiality.

????

????

QUESTION 9 OF 11

lines 57–58 (“At . . .enters”) (“At this point Julius and Danny drop out of our story, and Young Thomas enters.”) serves mainly to????

????????

A introduce a new character and setting to the story. B show the resulting impact of the rumor Julius started. C add a surprising twist to the story. D create a sense of suspense for the outcome of the prank.

QUESTION 5 OF 11

The conversation between Julius and Dan serves mainly to????????

????????

A show how bored Julius is with the inhabitants of Valley View. B provide a context for the relationships in the community. C demonstrate Dan’s gullibility when it comes to rumors. D establish why Julius’ story was so widely accepted.

????

QUESTION 6 OF 11

Which choice provides the best evidence for the answer to the previous question???

??

A lines 3–5 (“He . . . repeatedly”)(“He had not succeeded in \??

??

B lines 33–34 (“Julius . . . countenance”)(“Julius delivered himself of this tremendous lie with a transparently earnest countenance.”) C lines 45–49 (“Young Thomas . . . them”)(“Young Thomas was a con?rmed bachelor of ?fty, and Adelia Williams was forty; they were not supposed to be even well acquainted, as the Everetts and the Williamses had never been very friendly, although no open feud existed between them.”) D lines 50–52 (“Nevertheless . . . further”)(“Nevertheless, in view of Julius's circumstantial statements, the amazing news must be true, and Dan was instantly agog to carry it further.”) QUESTION 10 OF 11

Eben’s comments in lines 66–70 (“Ah . . . wife”) (“\off like that, Tom,\I was glad to hear it, although I was mighty surprised. I never thought of you and Adelia. But she's a ?ne little woman and will make you a capital wife.””) primarily indicate that he????????

????????

A disagrees strongly with Thomas. B ?nds the rumor to be funny. C believes that Thomas is lying. D feels unsurprised by the match.

????

QUESTION 11 OF 11

It can be inferred that Tom is \????????

????????

A feels exasperated by Julius’ tendency to gossip. B believes Adelia doesn’t want to marry him. C is agitated that he is the subject of a rumor. D has too many errands to run in Valley View.

????

QUESTION 7 OF 11

In the passage, Dan is characterized as someone who is????????

????????

A honest. B foolish. C stubborn. D distrustful.

QUESTION 8 OF 11

Which choice provides the best evidence for the answer to the previous question?

????????????????????????????

??????????????????????????????????????????

Exercise 355This passage is excerpted from Kate Chopin, \originally published in 1899.Line5101520253035404550\Robert, coming out on the porch where she was. Of course Edna would like to hear Mademoiselle Reisz play; but she feared it would be useless to entreat her.\She likes you. She will come.\to one of the far cottages, where Mademoiselle Reisz was shuf?ing away. She was dragging a chair in and out of her room, and at intervals objecting to the crying of a baby, which a nurse in the adjoining cottage was endeavoring to put to sleep. She was a disagreeable little woman, no longer young, who had quarreled with almost every one, owing to a temper which was self-assertive and a disposition to trample upon the rights of others. Robert prevailed upon her without any too great dif?culty.She entered the hall with him during a lull in the dance. She made an awkward, imperious little bow as she went in. She was a homely woman, with a small weazened face and body and eyes that glowed. She had absolutely no taste in dress, and wore a batch of rusty black lace with a bunch of arti?cial violets pinned to the side of her hair.\she requested of Robert. She sat perfectly still before the piano, not touching the keys, while Robert carried her message to Edna at the window. A general air of surprise and genuine satisfaction fell upon every one as they saw the pianist enter. There was a settling down, and a prevailing air of expectancy everywhere. Edna was a tri?e embarrassed at being thus signaled out for the imperious little woman's favor. She would not dare to choose, and begged that Mademoiselle Reisz would please herself in her selectionsEdna was what she herself called very fond of music. Musical strains, well rendered, had a way of evoking pictures in her mind. She sometimes liked to sit in the room of mornings when Madame Ratignolle played or practiced. One piece which that lady played Edna had entitled \was a short, plaintive, minor strain. The name of the piece was something else, but she called it \heard it there came before her imagination the ?gure of a man standing beside a desolate rock on the seashore. . . His attitude was one of hopeless resignation as he looked toward a distant bird winging its ?ight away from him.Another piece called to her mind a dainty young woman clad in an Empire gown, taking mincing dancing steps as she came down a long avenue between tall hedges. Again, another reminded her of children at play, and still another of nothing on earth but a demure lady stroking a cat.The very ?rst chords which Mademoiselle Reisz struck upon the piano sent a keen tremor down Mrs. Pontellier's spinal column.It was not the ?rst time she had heard an artist at the piano. Perhaps it was the ?rst time she was ready, 606570perhaps the ?rst time her being was tempered to take an impress of the abiding truth.She waited for the material pictures which she thought would gather and blaze before her imagination. She waited in vain. She saw no pictures of solitude, of hope, of longing, or of despair. But the very passions themselves were aroused within her soul, swaying it, lashing it, as the waves daily beat upon her splendid body. She trembled, she was choking, and the tears blinded her.Mademoiselle had ?nished. She arose, and bowing her stiff, lofty bow, she went away, stopping for neither thanks nor applause. As she passed along the gallery she patted Edna upon the shoulder.\woman was unable to answer; she pressed the hand of the pianist convulsively. Mademoiselle Reisz perceived her agitation and even her tears. She patted her again upon the shoulder as she said:\Bah!\toward her room.QUESTION 1 OF 11Over the course of the passage, the main focus shifts from???????????????? A a description of one woman to a detailed characterization of a group of people. B a depiction of a place to a statement of one character’s opinions about that place. C opinions shared by many laypeople to opinions held by a great artist. D the setup of a situation to one character’s reaction to that situation. QUESTION 2 OF 11In paragraph 4 (lines 22–28 (“\the words “satisfaction” and “expectancy” serve mainly to???????????????? A show that those in attendance have ?xed ideas about Mademoiselle Reisz’s performance. B characterize Mademoiselle Reisz’s feelings about being asked to play for the group. C indicate that Edna has been waiting for Mademoiselle Reisz to acknowledge her presence. D demonstrate that the listeners anticipate that Mademoiselle Reisz will not be able to play the pieces well. QUESTION 3 OF 11The primary purpose of paragraphs 2 and 3 (lines 5–21(“\???? A characterize Mademoiselle Reisz as someone who delights in making people uncomfortable solely for her own amusement. B emphasize that Madame Reisz has changed a great deal in the time Edna has known her. C describe Mademoiselle Reisz’s interaction with Robert, which re?ects her general interest in social pleasantries. D establish Mademoiselle Reisz’s unpleasant attitude and ????????????????????????????????????????

??????????????????????????????????????????

appearance, which contrast with the beautiful music she makes.

QUESTION 4 OF 11

It can be reasonably inferred that Edna refuses to choose the music that Mademoiselle Reisz will play because she????????

????????

A feels that her musical education is insuf?cient for the honor of making the selection. B is uncomfortable with being treated differently than the other guests. C does not know what music the other guests would prefer to hear. D wants to hear “Solitude” but doesn’t know how to ask for it.

????????

????????

A a member of the group. B someone about whom Mademoiselle Ratignolle is singing. C an imaginary woman. D Edna as a child.

QUESTION 9 OF 11

It can be reasonably inferred that Mademoiselle Reisz considers Edna to be worth playing for because????????

????????

A only Edna has received enough training to fully appreciate the music. B Edna feels passionately and deeply about the music. C Edna has never had the opportunity to hear her play before. D only Edna understands how it feels to be a musician.

QUESTION 5 OF 11

Which choice provides the best evidence for the answer to the previous question???

??

A lines 28–30 (“Edna . . . favor”)(“Edna was a tri?e

embarrassed at being thus signaled out for the imperious little woman's favor.”) ??

??

B line 32 (“Edna . . . music”)(“Edna was what she herself called very fond of music.”) C lines 34–35 (“She . . . practiced”)(“She sometimes liked to sit in the room of mornings when Madame Ratignolle played or practiced.”) D lines 35–37 (“One . . . strain”)(“ One piece which that lady played Edna had entitled \plaintive, minor strain.”) QUESTION 10 OF 11

Which choice provides the best evidence for the answer to the previous question???

??

A lines 50–51 (“It was . . . piano”)(“It was not the ?rst time she had heard an artist at the piano.”) ??

??

B lines 54–55 (“She . . . imagination”)(“She waited for the material pictures which she thought would gather and blaze before her imagination.”) C lines 63–64 (“As she . . . shoulder”)(“As she passed along the gallery she patted Edna upon the shoulder.”) D lines 67–68 (“Mademoiselle . . . tears”)(“Mademoiselle Reisz perceived her agitation and even her tears.”) ????

????

????

??

QUESTION 6 OF 11

As used in line 33 (“rendered”), “rendered” most nearly means????????

????????

A covered B made C provided D performed

??

QUESTION 11 OF 11

The passage suggests that Edna views Mademoiselle Reisz as someone who is????????

????????

A intimidating but profound. B cold and uninviting. C motherly and inspiring. D misguided but irresponsible.

QUESTION 7 OF 11

The imagery in paragraph 5 (lines 38–42 (“When she heard it there came before her imagination the ?gure of a man standing beside a desolate rock on the seashore. . . His attitude was one of hopeless resignation as he looked toward a distant bird winging its ?ight away from him.”) ) mainly serves to????????

????????

A show that Edna’s imagination is not in?uenced by her surroundings. B demonstrate Edna’s inability to distinguish reality from fantasy. C characterize the vivid scenes music brings to Edna’s mind. D make it clear that Edna longs to be alone in an isolated environment.

QUESTION 8 OF 11

The “dainty young woman clad in an Empire gown” (lines 43–44

(“a dainty young woman clad in an Empire gown”) ) is best understood to be

????????????????????????????

??????????????????????????????????????????

Exercise 455This passage is excerpted from Louisa May Alcott, Rose in Bloom, originally published in 1876.Line5101520253035404550Three young men stood together on a wharf one bright October day awaiting the arrival of an ocean steamer with an 60impatience which found a vent in lively skirmishes with a small lad, who pervaded the premises like a will-o'-the-wisp and afforded much amusement to the other groups assembled there.\65been abroad several years with her uncle, the doctor,\whispered one lady to another as the handsomest of the young men touched his hat to her as he passed, lugging the boy, whom he had just rescued from a little expedition down among the piles.70\\promising of the seven, but a little fast, people say,\answered the ?rst speaker with a shake of the head. \others his brothers?\75\man. He has just gone into business with the merchant uncle and bids fair to be an honor to his family. The other, with the eyeglasses and no gloves, is Mac, the odd one, just out of college.\\\pet of the whole family. Mercy on us he'll be in if they don't hold on to him!\The ladies' chat came to a sudden end just there, for by the time Jamie had been ?shed out of a hogshead, the steamer hove in sight and everything else was forgotten. As it swung slowly around to enter the dock, a boyish voice shouted, \Cousin Rose!\by Jamie as he stood on a post waving his arms like a windmill while his brother held onto the tail of his jacket.Yes, there they were, Uncle Alec swinging his hat like a boy, with Phebe smiling and nodding on one side and Rose kissing both hands delightedly on the other as she recognized familiar faces and heard familiar voices welcoming her home.\Madonna doesn't she? with that blue cloak round her, and her bright hair ?ying in the wind!\watched the group upon the deck with eager eyes.\much, but Phebe has. Why, she's a regular beauty!\Archie, staring with all his might at the dark-eyed young woman with the brilliant color and glossy black braids shining in the sun.\was all Mac said, but he was not looking at \as he made the fervent remark, for he saw only the slender blond girl nearby and stretched out his hands to meet hers, forgetful of the green water tumbling between them.During the confusion that reigned for a moment as the steamer settled to her moorings, Rose looked down into the four faces upturned to hers and seemed to read in them something that both pleased and pained her. It was only a glance, and her own eyes were full, but through the mist of happy tears she received the impression that Archie was about the same, that Mac had decidedly improved, and that something was amiss with Charlie. There was no time for observation, however, for in a moment the shoreward rush began, and before she could grasp her traveling bag, Jamie was clinging to her like an ecstatic young bear. She was with dif?culty released from his embrace to fall into the gentler ones of the elder cousins, who took advantage of the general excitement to welcome both blooming girls with affectionate impartiality. Then the wanderers were borne ashore in a triumphal procession, while Jamie danced rapturous jigs before them even on the gangway.Archie remained to help his uncle get the luggage through the Custom House, and the others escorted the damsels home. No sooner were they shut up in a carriage, however, than a new and curious constraint seemed to fall upon the young people, for they realized, all at once, that their former playmates were men and women now.QUESTION 1 OF 11Which choice best summarizes the passage????????????????? A The passage provides an explanation of relationships within a family. B The passage presents a revelation about children growing older. C The passage captures a conversation regarding reuniting cousins. D The passage describes an anecdote about traveling by ship. QUESTION 2 OF 11Over the course of the passage, the attitude of the cousins shifts from???????????????? A preoccupation to constraint. B apprehension to embarrassment. C anticipation to reticence. D contemplation to dif?dence. QUESTION 3 OF 11Which statement best characterizes the relationship among the reunited cousins????????????????? A They are excited to get back to their ordinary lives and friendships. B They have changed too dramatically to ?nd anything in common. C The younger cousins help put the older cousins at ease. D Their growing up has put limitations on their relationships. ????????????????????????????

??????????????????????????????????????????

QUESTION 4 OF 11

As used in line 3 (“vent”) , “vent” most nearly means????????

????????

A opening. B outlet. C crack. D passage.

????????

????????

A not actually paying attention to his uncle. B unaware of the possibility of falling in the ocean. C more excited to see his uncle than he is to see Rose. D distracted by all the excitement in the crowd.

QUESTION 9 OF 11

It can be inferred that Rose is \views her cousins from the ship because she????????

????????

A feels exhausted and confused after a long journey by sea. B believes her cousins don’t recognize her anymore. C senses that her relationship with her cousins has changed. D concludes that her cousins’ lives have gotten better without her.

QUESTION 5 OF 11

The conversation between the lady and the stranger in lines 7–26(“\

abroad several years with her uncle, the doctor,\another as the handsomest of the young men touched his hat to her as he passed, lugging the boy, whom he had just rescued from a little expedition down among the piles.”) (“They . . . him”) serves primarily to????????

????????

A show how the town locals tend to gossip about the family. B establish the relationships among the reuniting family. C introduce the traits of the family members who are waiting. D demonstrate the lady’s lack of knowledge about the family.

QUESTION 10 OF 11

Which choice provides the best evidence for the answer to the previous question???

??

A lines 57–61(“It was only a glance, and her own eyes were full, but through the mist of happy tears she received the impression that Archie was about the same, that Mac had decidedly improved, and that something was amiss with Charlie.”)(“It . . . Charlie”) B lines 64–68(“ She was with dif?culty released from his embrace to fall into the gentler ones of the elder cousins, who took advantage of the general excitement to welcome both blooming girls with affectionate impartiality. ”)(“She . . . impartiality”) C lines 61–64(“ There was no time for observation,

however, for in a moment the shoreward rush began, and before she could grasp her traveling bag, Jamie was clinging to her like an ecstatic young bear.”)(“There . . . bear”) D lines 73–76(“No sooner were they shut up in a carriage, however, than a new and curious constraint seemed to fall upon the young people, for they realized, all at once, that their former playmates were men and women now.”)(“No . . . now”)

QUESTION 6 OF 11

Which choice provides the best evidence for the answer to the previous question???

??

A lines 7–8(“They are the Campbells, waiting for their

cousin, who has been abroad several years with her uncle, the doctor,\(“They . . . doctor”) B lines 14–15(“\the most promising of the seven, but a little fast, people say,\(“Prince . . . say”) C lines 18–20(“The elder is Archie, a most exemplary young man. He has just gone into business with the merchant uncle and bids fair to be an honor to his family. ”)(“The . . . family”) D lines 24–26(“\Archibald, and the pet of the whole family. Mercy on us he'll be in if they don't hold on to him”)(“Oh . . . him”)

??

??

????

????

????

????

????

QUESTION 7 OF 11

Archie’s comments in lines 44–45 (“\that. Rose hasn't changed much, but Phebe has. Why, she's a regular beauty!””) (“Madonnas . . . beauty”) primarily indicate that he????????

????????

A disagrees strongly with Charlie. B ?nds Phebe to be beautiful. C thinks Rose looks like a Madonna. D feels distracted by Rose and Phebe.

QUESTION 11 OF 11

As used in lines 67–68 (“affectionate impartiality”), the phrase “affectionate impartiality” most nearly means????????

????????

A restraint. B aloofness. C tenderness. D enthusiasm.

QUESTION 8 OF 11

The description (lines 50–53(“ was all Mac said, but he was not looking at \slender blond girl nearby and stretched out his hands to meet hers, forgetful of the green water tumbling between them.”) ) that follows Mac's comment in line 49 (“\have him back?””) (\

????????????????????????????

??????????????????????????????????????????

Exercise 555This passage is excerpted from Jules Verne, Around the World in Eighty Days. Originally published in 1873.Line5101520253035404550The mansion in Saville Row, though not sumptuous, was exceedingly comfortable. The habits of its occupant were such as to demand but little from the sole domestic, but Phileas Fogg required him to be almost superhumanly prompt and regular. On this very 2nd of October he had dismissed James Forster, because that luckless youth had brought him shaving-water at eighty-four degrees Fahrenheit instead of eighty-six; and he was awaiting his successor, who was due at the house between eleven and half-past.Phileas Fogg was seated squarely in his armchair, his feet close together like those of a grenadier on parade, his hands resting on his knees, his body straight, his head erect; he was steadily watching a complicated clock which indicated the hours, the minutes, the seconds, the days, the months, and the years. At exactly half-past eleven Mr. Fogg would, according to his daily habit, quit Saville Row, and repair to the Reform1.A rap at this moment sounded on the door of the cosy apartment where Phileas Fogg was seated, and James Forster, the dismissed servant, appeared.\A young man of thirty advanced and bowed.\your name is John?\\Passepartout, a surname which has clung to me because I have a natural aptness for going out of one business into another. I believe I'm honest, monsieur, but, to be outspoken, I've had several trades. I've been an itinerant singer, a circus-rider, when I used to vault like Leotard,2 and dance on a rope like Blondin.3 Then I got to be a professor of gymnastics, so as to make better use of my talents; and then I was a sergeant ?reman at Paris, and assisted at many a big ?re. But I quitted France ?ve years ago, and, wishing to taste the sweets of domestic life, took service as a valet here in England. Finding myself out of place, and hearing that Monsieur Phileas Fogg was the most exact and settled gentleman in the United Kingdom, I have come to monsieur in the hope of living with him a tranquil life, and forgetting even the name of Passepartout.\\recommended to me; I hear a good report of you. You know my conditions?\\\\drawing an enormous silver watch from the depths of his pocket.\\\mention the error. Now from this moment, twenty-nine 60minutes after eleven, a.m., this Wednesday, 2nd October, you are in my service.\Phileas Fogg got up, took his hat in his left hand, put it on his head with an automatic motion, and went off without a word.Passepartout heard the street door shut once; it was his new master going out. He heard it shut again; it was his predecessor, James Forster, departing in his turn. Passepartout remained alone in the house in Saville Row.1A private members’ club in London2A French acrobat3A French tightrope walker and acrobatQUESTION 1 OF 10Over the course of the passage, the main focus shifts from???????????????? A a description of one character to an illustration of that character’s interactions with another character. B a characterization of the upper classes to an exposure of problems with the social order. C a depiction of a particular place and time to a prediction about one man’s future. D opinions held by an employer to the views asserted by a potential employee. QUESTION 2 OF 10The narrator describes the circumstances under which James Forster was ?red most likely in order to???????????????? A emphasize the standards that Phileas Fogg demands from his employees. B demonstrate the unfairness of Phileas Fogg’s methods. C represent social conditions that Phileas Fogg perpetuates. D show Phileas Fogg’s random acts of cruelty. QUESTION 3 OF 10In paragraph two (lines 10–17(“Phileas Fogg was seated squarely in his armchair, his feet close together like those of a grenadier on parade, his hands resting on his knees, his body straight, his head erect; he was steadily watching a complicated clock which indicated the hours, the minutes, the seconds, the days, the months, and the years. At exactly half-past eleven Mr. Fogg would, according to his daily habit, quit Saville Row, and repair to the Reform.*”)), the narrator characterizes Mr. Fogg as someone who???????????????? A lacks friends and unwillingly spends most of his time in solitude. B has become jaded and predictable after time spent in the military. C has high standards for both comportment and punctuality. D becomes bored easily because of his lack of interests. QUESTION 4 OF 10As used in line 22(“ advanced”), “advanced” most nearly means???????? A promoted. B approached. ????????????????????????????

??????????????????????????????????????????

????

????

C loaned. D supported.

????????

????????

A line 49(“\(“You . . . Fogg”) B line 50(“\(“Pardon . . . impossible”) C line 51(“\(“You . . . slow”) D lines 51–52(“No matter; it's enough to mention the error.”)(“No . . . error”)

QUESTION 5 OF 10

In the seventh paragraph (lines 33–40(“But I quitted France ?ve years ago, and, wishing to taste the sweets of domestic life, took service as a valet here in England. Finding myself out of place, and hearing that Monsieur Phileas Fogg was the most exact and settled gentleman in the United Kingdom, I have come to monsieur in the hope of living with him a tranquil life, and forgetting even the name of Passepartout.\the words “sweets,” “settled,” and “tranquil” primarily serve to????????

????????

A characterize Passepartout’s past. B reveal Passepartout’s harsh and manipulative side. C describe Passepartout’s approach to all of his career choices. D describe Passepartout’s ideal work environment.

QUESTION 10 OF 10

In the ?nal two paragraphs (lines 55–61(“Phileas Fogg got up, took his hat in his left hand, put it on his head with an automatic motion, and went off without a word.”)), the description of Mr. Fogg’s departure mainly serves to????????

????????

A show that Passepartout already has full control over Fogg’s household. B demonstrate Fogg’s rudeness towards anyone of a lower class. C highlight the plight of the previous servant. D emphasize the abrupt decision Fogg has made to hire Passepartout.

QUESTION 6 OF 10

Mr. Fogg's \inferred to include????????

????????

A the exacting and precise standards he expects of his employees. B his employee’s willingness to take on a variety of different jobs. C excellent references from former employers. D a name that he ?nds suitable.

QUESTION 7 OF 10

Which choice provides the best evidence for the answer to the previous question???

??

A lines 4–5(“Phileas Fogg required him to be almost superhumanly prompt and regular.”)(“Phileas . . . regular”) B lines 26–28(“I have a natural aptness for going out of one business into another.”)(“I . . . another”) C line 41(“\(“Passepartout . . . me”) D lines 41–42(“\good report of you. ”)(“You are . . . you”)

Find more here at AAA studio

????

????

????

QUESTION 8 OF 10

At the end of the passage, the discussion about the time suggests that Mr. Fogg’s attitude towards his new employee will be????????

????????

A strict and harshly unforgiving. B exacting but initially lenient. C pleasant and constantly relaxed. D annoyed but frequently fatherly.

QUESTION 9 OF 10

Which choice provides the best evidence for the answer to the previous question?

??????????????????????????????

??????????????????????????????????????????

ANSWER KEY

Exercise1Exercise2Exercise3Exercise4Exercise51234567891011CACBDBABACA1234567891011ACABDDAADCC1234567891011DADBADCCBDA1234567891011BCDBBABACDA12345678910AACBDAABDDFind more here at AAA studio

??????????????????????????????

Reading Practices for the Redesigned SAT 1.2 Literature Level 3

Find more here at AAA studio

Proudly presented by Khan Academy Humbly brought to you by AAA Studio

??????????????????????????????????????????

Exercise 1This passage is excerpted from Charlotte Bronte, Villette. Originally published in 1853. In this chapter, the narrator and her host, Mrs. Bretton, are trying to occupy a young girl, Paulina, who is staying with them.55Line5101520253035404550One afternoon, Mrs. Bretton, coaxing [Paulina] from her usual station in a corner, had lifted her into the window-seat, and, by way of occupying her attention, told her to watch the passengers and count how many ladies should go down the street in a given time. She had sat listlessly, hardly looking, and not counting, when—my eye being ?xed on hers—I witnessed in its iris and pupil a startling trans?guration. These sudden, dangerous natures—sensitive as they are called—offer many a curious spectacle to those whom a cooler temperament has secured from participation in their angular vagaries. The ?xed and heavy gaze swum, trembled, then glittered in ?re; the small, overcast brow cleared; the trivial and dejected features lit up; the sad countenance vanished, and in its place appeared a sudden eagerness, an intense expectancy. \is!\Like a bird or a shaft, or any other swift thing, she was gone from the room. How she got the house-door open I cannot tell; probably it might be ajar; perhaps Warren was in the way and obeyed her behest, which would be impetuous enough. I—watching calmly from the window—saw her, in her black frock and tiny braided apron (to pinafores she had an antipathy), dart half the length of the street; and, as I was on the point of turning, and quietly announcing to Mrs. Bretton that the child was run out mad, and ought instantly to be pursued, I saw her caught up, and rapt at once from my cool observation, and from the wondering stare of the passengers. A gentleman had done this good turn, and now, covering her with his cloak, advanced to restore her to the house whence he had seen her issue.I concluded he would leave her in a servant's charge and withdraw; but he entered: having tarried a little while below, he came up-stairs.His reception immediately explained that he was known to Mrs. Bretton. She recognised him; she greeted him, and yet she was ?uttered, surprised, taken unawares. Her look and manner were even expostulatory; and in reply to these, rather than her words, he said,—\found it impossible to leave the country without seeing with my own eyes how she settled.\\\This question he addressed to Paulina, as he sat down and placed her gently on the ground before him.\knee, and gazed up into his face.It was not a noisy, not a wordy scene: for that I was thankful; but it was a scene of feeling too brimful, and which, because the cup did not foam up high or furiously over?ow, only oppressed one the more. On all occasions of vehement, unrestrained expansion, a sense of disdain or ridicule comes 6065to the weary spectator's relief; whereas I have ever felt most burdensome that sort of sensibility which bends of its own will, a giant slave under the sway of good sense.Mr. Home was a stern-featured—perhaps I should rather say, a hard-featured man: his forehead was knotty, and his cheekbones were marked and prominent. The character of his face was quite Scotch; but there was feeling in his eye, and emotion in his now agitated countenance. His northern accent in speaking harmonised with his physiognomy. He was at once proud-looking and homely-looking. He laid his hand on the child's uplifted head. She said—\He kissed her. I wished she would utter some hysterical cry, so that I might get relief and be at ease. She made wonderfully little noise: she seemed to have got what she wanted—all she wanted, and to be in a trance of content. Neither in mien nor in features was this creature like her sire, and yet she was of his strain: her mind had been ?lled from his, as the cup from the ?agon.QUESTION 1 OF 11Over the course of the passage, the main focus shifts from???????????????? A a critique of the treatment of children to a depiction of a particular child’s situation. B observations of a character’s mental state to a domestic reunion. C the opinions stated by one character to the opposing views asserted by another. D the description of a character’s past to a prediction about the character’s future. QUESTION 2 OF 11In the passage, the narrator characterizes Paulina as someone who goes from???????????????? A dissatis?ed to content. B confused to understanding. C angry to experiencing catharsis. D contented to ecstatic. QUESTION 3 OF 11In the ?rst paragraph, the words “swum,” “trembled,” and “glittered” primarily serve to???????????????? A describe the atmosphere Mr. Home’s presence creates in the room. B reveal that Paulina has become upset by what she has seen. C emphasize Paulina’s excitement at what she has seen. D characterize the effect Paulina’s boredom has had on her appearance. QUESTION 4 OF 11According to the passage, Paulina cries, “It is!” (line 15 (“\because???? A she is trying to entertain herself. ????????????????????????????

??????????????????????????????????????????

??????

??????

B she perceives the narrator’s gaze as a challenge. C she has made a mistake she refuses to recognize. D she has recognized her father on the street.

????

????

C Mr. Home has come for a regularly-scheduled visit with Paulina. D Mrs. Bretton was not expecting to see Mr. Home.

QUESTION 5 OF 11

In lines 16–17 (“Like a bird or a shaft, or any other swift thing, she was gone from the room.”), the description of Paulina’s disappearance mainly serves to????????

????????

A show a physical response from Paulina that parallels her emotional response. B demonstrate how the narrator’s reaction to the scene outside affects Paulina. C show the lack of security in the household and why it is not suitable for Paulina. D highlight Paulina’s excitable and easily dissatis?ed nature.

QUESTION 10 OF 11

Which choice provides the best evidence for the answer to the previous question???

??

A lines 34–35 (“She . . . unawares”)(“She recognised him; she greeted him, and yet she was ?uttered, surprised, taken unawares.”) B lines 35–36 (“Her . . . expostulatory”)(“Her look and manner were even expostulatory;”) C line 40 (“But . . . her”)(“\ D line 41 (“And . . . Polly”)(“And how is papa's little Polly?\????

????

????

QUESTION 6 OF 11

As used in line 22 (“dart”), “dart” most nearly means????????

????????

A sew. B shoot. C panic. D run.

QUESTION 11 OF 11

The description of Mr. Home in paragraph 10 (lines 54–61

(“Mr. Home was a stern-featured—perhaps I should rather say, a hard-featured man: his forehead was knotty, and his cheekbones were

marked and prominent. The character of his face was quite Scotch; but there was feeling in his eye, and emotion in his now agitated

countenance. His northern accent in speaking harmonised with his physiognomy. He was at once proud-looking and homely-looking. He laid his hand on the child's uplifted head. She said—\) primarily serves to????????

????????

A demonstrate the duality of his character. B emphasize his hard, cold nature. C show the strong and obvious love he has for Paulina. D reinforce his state of confusion.

QUESTION 7 OF 11

Based on the narrator’s initial reaction to Mr. Home, it can be reasonably inferred that the narrator assumes Mr. Home is????????

????????

A Paulina’s father. B an acquaintance of Mrs. Bretton. C a stranger. D a servant named Warren.

QUESTION 8 OF 11

Which choice provides the best evidence for the answer to the previous question???

??

A lines 18–20 (“Perhaps . . . enough”)(“perhaps Warren was in the way and obeyed her behest, which would be impetuous enough.”) B lines 30–31 (“I . . . withdraw”)(“I concluded he would leave her in a servant's charge and withdraw;”) C lines 33–34 (“His . . .Mrs. Bretton”)(“His reception

immediately explained that he was known to Mrs. Bretton.”) D lines 44–45 (“How . . . face”)(“\the reply, as she leaned on his knee, and gazed up into his face.”) Find more here at

AAA studio

????

????

????

QUESTION 9 OF 11

Based on the description of Mr. Home’s encounter with Mrs. Bretton, it can reasonably be inferred that????

????

A Mrs. Bretton has attempted to prevent Mr. Home from seeing Paulina. B Mr. Home feels comfortable enough with Mrs. Bretton to address her by a nickname.

????????????????????????????

??????????????????????????????????????????

Exercise 255This passage is adapted from Mark Twain, \published in 1872.Line5101520253035404550My brother had just been appointed Secretary of Nevada Territory—an of?ce of such majesty that it concentrated in itself the duties and dignities of Treasurer, Comptroller, Secretary of State, and Acting Governor in the Governor's absence. A salary of eighteen hundred dollars a year and the title of \wild and imposing grandeur. I was young and ignorant, and I envied my brother. I coveted his distinction and his ?nancial splendor, but particularly and especially the long, strange journey he was going to make, and the curious new world he was going to explore. He was going to travel! I never had been away from home, and that word \charm for me. Pretty soon he would be hundreds and hundreds of miles away on the great plains and deserts, and among the mountains of the Far West, and would see buffaloes and Indians, and prairie dogs, and antelopes, and have all kinds of adventures, and have ever such a ?ne time, and write home and tell us all about it, and be a hero. And he would see the gold mines and the silver mines, and maybe go about of an afternoon when his work was done, and pick up two or three pailfuls of shining slugs, and nuggets of gold and silver on the hillside. And by and by he would become very rich, and return home by sea, and be able to talk as calmly about San Francisco and the ocean, and \as if it was nothing of any consequence to have seen those marvels face to face.What I suffered in contemplating his happiness, pen cannot describe. And so, when he offered me, in cold blood, the sublime position of private secretary under him, it appeared to me that the heavens and the earth passed away, and the ?rmament was rolled together as a scroll! I had nothing more to desire. My contentment was complete.At the end of an hour or two I was ready for the journey. Not much packing up was necessary, because we were going in the overland stage from the Missouri frontier to Nevada, and passengers were only allowed a small quantity of baggage apiece. There was no Paci?c railroad in those ?ne times of ten or twelve years ago—not a single rail of it. I only proposed to stay in Nevada three months—I had no thought of staying longer than that. I meant to see all I could that was new and strange, and then hurry home to business. I little thought that I would not see the end of that three-month pleasure excursion for six or seven uncommonly long years!I dreamed all night about Indians, deserts, and silver bars, and in due time, next day, we took shipping at the St. Louis wharf on board a steamboat bound up the Missouri River.We were six days going from St. Louis to \that was so dull, and sleepy, and eventless that it has left no more impression on my memory than if its duration had been six minutes instead of that many days. No record is left in my mind, now, concerning it, but a confused jumble of savage-looking snags, which our boat deliberately walked over with 60one wheel* or the other; and of reefs which we butted and butted, and then retired from and climbed over in some softer place; and of sand-bars which we roosted on occasionally, and rested, and then got out our crutches and sparred over.In fact, the boat might almost as well have gone to St. Jo. by land, for she was walking most of the time, anyhow—climbing over reefs and clambering over snags patiently and laboriously all day long. The captain said she was a \boat, and all she wanted was more \wheel. I thought she wanted a pair of stilts, but I had the deep sagacity not to say so.*The narrator is on a paddle steamer, a boat that uses steam power to turn a large wheel in the rear of the boat.QUESTION 1 OF 11Over the course of the passage, the main focus shifts from???????????????? A the narrator’s view on his brother’s job to the narrator’s hopes for his own trip. B the narrator’s jealousy of his brother to the narrator’s anxiety about his own trip. C the narrator’s expectations about the Far West to the reality of life in the Far West. D the narrator’s excitement about traveling west to the narrator’s fear of leaving home. QUESTION 2 OF 11As used in line 2 (“majesty”), “majesty” most nearly means???????????????? A solemn royalty. B breathtaking beauty. C impressive dignity. D extreme dif?culty. QUESTION 3 OF 11Based on lines 7–8 (“I was young and ignorant, and I envied my brother.”), (“I . . . brother)” it can reasonably be inferred that the narrator’s perspective is one of???????????????? A critical re?ection upon his past views. B objective analysis of his prior actions. C fond recollection of his recent adventures. D regretful remembrance of his past mistakes. QUESTION 4 OF 11Which statement best characterizes the narrator’s relationship with his brother????????????????? A The narrator is disinterested in emulating his brother. B The narrator is solely motivated by his brother’s wealth. C The narrator is generally unrealistic about his brother’s situation. D The narrator is mostly critical of his brother’s recent decisions. QUESTION 5 OF 11Which choice provides the best evidence for the answer to the previous question????? A lines 1–5 (“My . . . absence”)(“My brother had just been appointed Secretary of Nevada Territory—an of?ce of such ????????????????????????????

??????????????????????????????????????????

majesty that it concentrated in itself the duties and dignities of Treasurer, Comptroller, Secretary of State, and Acting Governor in the Governor's absence”) ??

??

B lines 8–11 (“I . . . explore”)(“I coveted his distinction and his ?nancial splendor, but particularly and especially the long, strange journey he was going to make, and the curious new world he was going to explore.”) C lines 18–22 (“And . . . hillside”)(“And he would see the gold mines and the silver mines, and maybe go about of an afternoon when his work was done, and pick up two or three pailfuls of shining slugs, and nuggets of gold and silver on the hillside.”) D lines 27–28 (“What . . . describe”)(“What I suffered in contemplating his happiness, pen cannot describe.”) ??

??

D lines 41–43 (“I . . . years”)(“ I little thought that I would not see the end of that three-month pleasure excursion for six or seven uncommonly long years”) ????

QUESTION 9 OF 11

Which situation is most similar to the situation described in paragraph 4 (lines 44–46 (“I dreamed all night about Indians, deserts, and silver bars, and in due time, next day, we took shipping at the St. Louis wharf on board a steamboat bound up the Missouri River.”))?????????

????????

A While paying his bills, a man daydreams about winning money in the lottery. B The night before the school play, a boy worries that he will forget his lines. C On her way to practice, a softball player pictures herself going up to bat. D As he prepares for a diving excursion, a diver imagines ?nding sunken treasure.

????

QUESTION 6 OF 11

At the end of the ?rst paragraph, the description of the brother’s return mainly serves to????????

????????

A demonstrate that the narrator is correct to be envious of his brother. B call into question the idea that the Far West is a “curious new world.” C suggest that the narrator’s expectations about the Far West are fantastical. D support the narrator’s view of his brother’s job with speci?c examples of its glamour.

QUESTION 10 OF 11

As used in line 55 (“roosted”), “roosted on” most nearly means????????

????????

A made a nest. B lay down to sleep. C settled in a single place. D climbed to the top of.

QUESTION 11 OF 11

In the description of the boat's movements, the words “walked” and “walking” primarily serve to establish a tone of????????

????????

A humor. B foreboding. C strangeness. D awe.

QUESTION 7 OF 11

It can be reasonably inferred from the discussion of Nevada in

paragraph 3 (lines 33–43 (“At the end of an hour or two I was ready for the journey. Not much packing up was necessary, because we were going in the overland stage from the Missouri frontier to Nevada, and passengers were only allowed a small quantity of baggage apiece. There was no Paci?c railroad in those ?ne times of ten or twelve years ago—not a single rail of it. I only proposed to stay in Nevada three

months—I had no thought of staying longer than that. I meant to see all I could that was new and strange, and then hurry home to business. I little thought that I would not see the end of that three-month pleasure excursion for six or seven uncommonly long years!”)) that the narrator's stay was????????

????????

A less exciting than he had expected. B much longer than he had planned. C more pleasurable than he had anticipated. D more dangerous than he had expected.

Find more here at

AAA studio

QUESTION 8 OF 11

Which choice provides the best evidence for the answer to the previous question???

??

A lines 34–37 (“Not . . . apiece”)(“Not much packing up was necessary, because we were going in the overland stage from the Missouri frontier to Nevada, and passengers were only allowed a small quantity of baggage apiece.”) B lines 38–40 (“I only . . . that”)(“I only proposed to stay in Nevada three months—I had no thought of staying longer than that.”) C lines 40–41 (“I meant . . . business”)(“ I meant to see all I could that was new and strange, and then hurry home to business.”) ????

????

????????????????????????????

??????????????????????????????????????????

Exercise 3This passage is excerpted from Nathaniel Hawthorne, The House of the 55Seven Gables. Originally published in 1851. In this scene, set in the American Colonies when they were still governed by England, Colonel Pyncheon holds a party at his home for a visiting English dignitary.Line5101520253035404550One inauspicious circumstance there was, which awakened a hardly concealed displeasure in the breasts of a few of the more punctilious visitors. The founder of this stately mansion—a gentleman noted for the square and ponderous courtesy of his demeanor, ought surely to have stood in his own hall, and to have offered the ?rst welcome to so many eminent personages as here presented themselves in honor of his solemn festival. He was as yet invisible; the most favored of the guests had not beheld him. This sluggishness on Colonel Pyncheon's part became still more unaccountable, when the second dignitary of the province made his appearance, and found no more ceremonious a reception. The lieutenant-governor, although his visit was one of the anticipated glories of the day, had alighted from his horse, and assisted his lady from her side-saddle, and crossed the Colonel's threshold, without other greeting than that of the principal domestic.This person—a gray-headed man, of quiet and most respectful deportment—found it necessary to explain that his master still remained in his study, or private apartment; on entering which, an hour before, he had expressed a wish on no account to be disturbed.\taking the servant aside, \lieutenant-governor? Summon Colonel Pyncheon at once! I know that he received letters from England this morning; and, in the perusal and consideration of them, an hour may have passed away without his noticing it. But he will be ill-pleased, I judge, if you suffer him to neglect the courtesy due to one of our chief rulers, and who may be said to represent King William, in the absence of the governor himself. Call your master instantly.\\perplexity, but with a backwardness that strikingly indicated the hard and severe character of Colonel Pyncheon's domestic rule; \and, as your worship knows, he permits of no discretion in the obedience of those who owe him service. Let who list open yonder door; I dare not, though the governor's own voice should bid me do it!\\governor, who had overheard the foregoing discussion, and felt himself high enough in station to play a little with his dignity. \that the good Colonel came forth to greet his friends; else we shall be apt to suspect that he has taken a sip too much of his Canary wine, in his extreme deliberation which cask it were best to broach in honor of the day! But since he is so much behindhand, I will give him a remembrancer myself!\Accordingly, with such a tramp of his ponderous riding-boots as might of itself have been audible in the remotest of 6065707580the seven gables, he advanced to the door, which the servant pointed out, and made its new panels reecho with a loud, free knock. Then, looking round, with a smile, to the spectators, he awaited a response. As none came, however, he knocked again, but with the same unsatisfactory result as at ?rst. And now, being a tri?e choleric in his temperament, the lieutenant-governor uplifted the heavy hilt of his sword, wherewith he so beat and banged upon the door, that, as some of the bystanders whispered, the racket might have disturbed the dead. Be that as it might, it seemed to produce no awakening effect on Colonel Pyncheon. When the sound subsided, the silence through the house was deep, dreary, and oppressive, notwithstanding that the tongues of many of the guests had already been loosened by a surreptitious cup or two of wine or spirits.\governor, whose smile was changed to a frown. \that our host sets us the good example of forgetting ceremony, I shall likewise throw it aside, and make free to intrude on his privacy.\He tried the door, which yielded to his hand, and was ?ung wide open by a sudden gust of wind that passed, as with a loud sigh, from the outermost portal through all the passages and apartments of the new house. It rustled the silken garments of the ladies, and waved the long curls of the gentlemen's wigs, and shook the window-hangings and the curtains of the bedchambers; causing everywhere a singular stir, which yet was more like a hush. A shadow of awe and half-fearful anticipation—nobody knew wherefore, nor of what—had all at once fallen over the company.QUESTION 1 OF 11Over the course of the passage, the main focus shifts from????????????????A the unusual behavior of a single character to a general sense of mystery. B the characterization of the party guests to the actions of the host. C a celebration of a certain social class to the denunciation of that class. D a description of a bygone era to a lament for the passing of that era. QUESTION 2 OF 11The narrator implies that the Colonel’s behavior is????????????????A eccentrically charming. B generally lenient. C unusually withdrawn. D overly servile. QUESTION 3 OF 11Which choice provides the best evidence for the answer to the previous question?????A lines 3–8 (“The founder . . . festival”)(“The founder of this stately mansion—a gentleman noted for the square and ponderous courtesy of his demeanor, ought surely to have stood in his own hall, and to have offered the ?rst welcome to so many eminent personages as here presented themselves in honor of his solemn festival.”) ????????????????????????????

??????????????????????????????????????????

??

??

B lines 9–13 (“This . . . reception”)(“This sluggishness on Colonel Pyncheon's part became still more unaccountable, when the second dignitary of the province made his

appearance, and found no more ceremonious a reception.”) C lines 18–22 (“This . . . disturbed”)(“This person—a gray-headed man, of quiet and most respectful deportment—found it necessary to explain that his master still remained in his study, or private apartment; on entering which, an hour before, he had expressed a wish on no account to be disturbed.”) D lines 36–38 (“my . . . service”)(“\exceeding strict; and, as your worship knows, he permits of no discretion in the obedience of those who owe him service.”) ????????

????????

A He wants to snub the lieutenant-governor. B He has fallen asleep. C He is still reading letters from England. D He has forgotten about the party.

????

QUESTION 8 OF 11

The passage’s account of the high sheriff’s behavior primarily serves to????????

????????

A add depth to a secondary character. B transition from the guests’ arrival to the discovery of the Colonel’s absence. C accentuate the gravity of the Colonel’s absence. D highlight existing class structures at work within the narrative.

????

QUESTION 4 OF 11

What can reasonably be inferred about the reason for the lieutenant-governor’s attitude?????????

????????

A He believes himself to be more valued in the political hierarchy than he actually is. B He is used to being treated with deference because of his position. C He is not intelligent enough to understand what is happening in the house. D The Colonel’s absence hurts his feelings, because they are old friends.

QUESTION 9 OF 11

As used in line 50 (“ponderous”), “ponderous” most nearly means????????

????????

A oppressive. B heavy. C troublesome. D thoughtful.

QUESTION 5 OF 11

Which choice provides the best evidence for the answer to the previous question???

??

A lines 13–17 (“The . . domestic”)(“The lieutenant-governor, although his visit was one of the anticipated glories of the day, had alighted from his horse, and assisted his lady from her side-saddle, and crossed the Colonel's threshold,

without other greeting than that of the principal domestic.”) B lines 28–32 (“But . . . himself”)(“But he will be ill-pleased, I judge, if you suffer him to neglect the courtesy due to one of our chief rulers, and who may be said to represent King William, in the absence of the governor himself.”) C lines 38–40 (“Let . . . it”)(“ Let who list open yonder door; I dare not, though the governor's own voice should bid me do it!\ D lines 41–44 (“Pooh . . . dignity”)(“\high sheriff!\

overheard the foregoing discussion, and felt himself high enough in station to play a little with his dignity.”) QUESTION 10 OF 11

What is the main purpose of the words “reecho,” “banged,” and

“racket,” used in the sixth paragraph (lines 50–61 (“Accordingly, with such a tramp of his ponderous riding-boots as might of itself have been audible in the remotest of the seven gables, he advanced to the door, which the servant pointed out, and made its new panels reecho with a loud, free knock. Then, looking round, with a smile, to the spectators, he awaited a response. As none came, however, he knocked again, but with the same unsatisfactory result as at ?rst. And now, being a tri?e choleric in his temperament, the lieutenant-governor uplifted the heavy hilt of his sword, wherewith he so beat and banged upon the door, that, as some of the bystanders whispered, the racket might have disturbed the dead.”))?????????

????????

A They demonstrate the turmoil caused by the lieutenant-governor. B They highlight the raucous nature of the Colonel’s party. C They characterize the lieutenant-governor as a primarily violent man. D They illustrate the differences between characterizations of the Colonel and the lieutenant-governor.

????

????

????

QUESTION 11 OF 11

The lieutenant-governor’s utterance in paragraph seven (lines 67–71(“\whose smile was changed to a frown. \the good example of forgetting ceremony, I shall likewise throw it aside, and make free to intrude on his privacy.\????????

????????

A summarize previous events. B characterize the Colonel’s attitude. C introduce a new phase in the story. D juxtapose the character’s words with his behavior.

QUESTION 6 OF 11

The actions of the Colonel’s servant can best be described as motivated by????????

????????

A his over-eagerness to please the Colonel. B the pretentiousness that his position requires. C his confusion over the high-sheriff’s request. D a sense of duty to his employer.

QUESTION 7 OF 11

According to the passage, what does the high sheriff give as the reason for the Colonel’s absence?

????????????????????????????

??????????????????????????????????????????

Exercise 455The following passage is adapted from Patrick Waddington, The Street That Got Mislaid, ?Patrick Waddington, 1954.Line5101520253035404550Marc Girondin had worked in the ?ling section of the city hall's engineering department for so long that the city was laid out in his mind like a map, full of names and places, intersecting streets and streets that led nowhere, blind alleys and winding lanes.In all Montreal no one possessed such knowledge; a dozen policemen and taxi drivers together could not rival him. That is not to say that he actually knew the streets whose names he could recite like a series of incantations, for he did little walking. He knew simply of their existence, where they were, and in what relation they stood to others.But it was enough to make him a specialist. He was undisputed expert of the ?ling cabinets where all the particulars of all the streets from Abbott to Zotique were indexed, back, forward and across. Those aristocrats, the engineers, the inspectors of water mains and the like, all came to him when they wanted some little particular, some detail, in a hurry. They might despise him as a lowly clerk, but they needed him all the same.Marc much preferred his of?ce, despite the profound lack of excitement of his work, to his room on Oven Street (running north and south from Sherbrooke East to St. Catherine), where his neighbors were noisy and sometimes violent, and his landlady consistently so. He tried to explain the meaning of his existence once to a fellow tenant, Louis, but without much success. Louis, when he got the drift, was apt to sneer.\so who cares? Why the excitement?\\live.\\\\get my mail here, don't I?\Marc shook his head patiently.\Street because it says so in my ?ling cabinet at city hall. The post of?ce sends you mail because my card index tells it to. If my cards didn't say so, you wouldn't exist and Oven Street wouldn't either. That, my friend, is the triumph of bureaucracy.\Louis walked away in disgust. \landlady,\So Marc continued on his undistinguished career, his fortieth birthday came and went without remark, day after day passed uneventfully. A street was renamed, another constructed, a third widened; it all went carefully into the ?les, back, forward and across.And then something happened that ?lled him with amazement, shocked him beyond measure, and made the world of the ?ling cabinets tremble to their steel bases.606570One August afternoon, opening a drawer to its fullest extent, he felt something catch. Exploring farther, he discovered a card stuck at the back between the top and bottom. He drew it out and found it to be an old index card, dirty and torn, but still perfectly decipherable. It was labeled RUE DE LA BOUTEILLE VERTE, or GREEN BOTTLE STREET.Marc stared at it in wonder. He had never heard of the place or of anything resembling so odd a name. Undoubtedly it had been retitled in some other fashion be?tting the modern tendency. He checked the listed details and ruf?ed con?dently through the master ?le of street names. It was not there. He made another search, careful and protracted, through the cabinets. There was nothing. Absolutely nothing.Once more he examined the card. There was no mistake. The date of the last regular street inspection was exactly ?fteen years, ?ve months and fourteen days ago.As the awful truth burst upon him, Marc dropped the card in horror, then pounced on it again fearfully, glancing over his shoulder as he did so.It was a lost, a forgotten street. For ?fteen years and more it had existed in the heart of Montreal, not half a mile from city hall, and no one had known. It had simply dropped out of sight, a stone in water.QUESTION 1 OF 11Over the course of the passage, the main focus shifts from????????????????A an explanation of a city’s layout to a description of one particular street. B the description of a character’s personality to the introduction of a life-changing event. C an interaction between two characters to the effects of that interaction on one of them. D a character’s thoughts to the depiction of him acting upon those thoughts. QUESTION 2 OF 11The imagery in lines 2–5 (“city…lanes”) (“city was laid out in his mind like a map, full of names and places, intersecting streets and streets that led nowhere, blind alleys and winding lanes.”) primarily serves to????????????????A characterize the mysterious nature of Montreal. B demonstrate the intricacy of Marc’s memory. C show that nobody else could understand the map. D emphasize Marc’s exceptional sense of direction. QUESTION 3 OF 11The situation described in paragraph two (lines 6–11 (“In all Montreal no one possessed such knowledge; a dozen policemen and taxi drivers together could not rival him. That is not to say that he actually knew the streets whose names he could recite like a series of incantations, for he did little walking. He knew simply of their existence, where they were, and in what relation they stood to others.”)) is most like????????A a cook who is an expert on a speci?c dish but who has never tasted it. B a travel agent who makes a living describing destinations to clients. ????????????????????????????

??????????????????????????????????????????

????

????

C a librarian who catalogues and organizes books for the general public. D a student who is interested in a particular artist but never seen any of her work.

????????

????????

A confused as to why he forgot to ?le a particular street. B curious about what the street contains. C anxious about the quality of the street inspections. D terri?ed by the discovery he has made.

QUESTION 4 OF 11

Which statement best characterizes Marc’s relationship with his job?????????

????????

A He enjoys his job but dislikes certain tasks he must perform. B He is good at his job but does not enjoy it. C He is consumed by his job and believes it to be all-important. D He ?nds his job fun but does not realize how signi?cant his work is.

QUESTION 10 OF 11

Which choice provides the best evidence for the answer to the previous question???

??

A lines 64–66 (“It . . . nothing”)(“It was not there. He made another search, careful and protracted, through the cabinets. There was nothing. Absolutely nothing.”) B lines 68–69 (“The date . . . ago”)(“The date of the last regular street inspection was exactly ?fteen years, ?ve months and fourteen days ago.”) C lines 70–72 (“As . . . so”)(“As the awful truth burst upon him, Marc dropped the card in horror, then pounced on it again fearfully, glancing over his shoulder as he did so.”) D line 73 (“It . . . street”)(“It was a lost, a forgotten street.”) ????

QUESTION 5 OF 11

Based on the passage, the other workers in the engineering department view Marc as????????

????????

A valuable but looked-down-upon. B intelligent but underused. C superior but shy. D inferior but quick.

????

????

QUESTION 6 OF 11

It can be reasonably inferred that Marc and Louis differ primarily because????????

????????

A Marc is poetic and Louis is unimaginative. B Marc is delusional and Louis is realistic. C Marc is productive and Louis is lazy. D Marc is kind and Louis is brusque.

QUESTION 11 OF 11

lines 73–76 (“It was . . . water”) (“It was a lost, a forgotten street. For ?fteen years and more it had existed in the heart of Montreal, not half a mile from city hall, and no one had known. It had simply dropped out of sight, a stone in water.”) mainly serve to????????

????????

A highlight the importance of a character’s discovery. B illustrate the main character’s unusual point of view. C describe a journey taken by the main character. D depict a new setting that a character will explore.

QUESTION 7 OF 11

Which choice provides the best evidence for the answer to the previous question?????

????

A lines 34–37 (“How . . . said”)(“\ain't I? I pay my rent, don't I? I get my mail here, don't I?\ B lines 37–39 (“You . . . hall”)(“\Street because it says so in my ?ling cabinet at city hall.”) C lines 39–40 (“The post . . . to”)(“The post of?ce sends you mail because my card index tells it to.”) D lines 40–44 (“If . . . muttered”)(“If my cards didn't say so, you wouldn't exist and Oven Street wouldn't either. That, my friend, is the triumph of bureaucracy.\Find more here at

AAA studio

????

????

QUESTION 8 OF 11

As used in line 54 (“Exploring”), “exploring” most nearly means????????

????????

A traveling. B inquiring. C examining. D evaluating.

QUESTION 9 OF 11

At the end of the passage, the narrator implies that Marc is

????????????????????????????

??????????????????????????????????????????

Exercise 555This passage is adapted from Sir Arthur Conan Doyle, The Adventures of Sherlock Holmes. Originally published in 1892. The narrator, Dr. Watson, works closely with detective Sherlock Holmes.Line5101520253035404550The portly client puffed out his chest with an appearance of some little pride and pulled a dirty and wrinkled newspaper from the inside pocket of his greatcoat. As he glanced down the advertisement column, with his head thrust forward and the paper ?attened out upon his knee, I took a good look at the man and endeavoured, after the fashion of my companion, to read the indications which might be presented by his dress or appearance.I did not gain very much, however, by my inspection. Our visitor bore every mark of being an average commonplace British tradesman, obese, pompous, and slow. He wore rather baggy grey shepherd’s check trousers, a not over-clean black frock-coat, unbuttoned in the front, and a drab waistcoat with a heavy brassy Albert chain, and a square pierced bit of metal dangling down as an ornament. A frayed top-hat and a faded brown overcoat with a wrinkled velvet collar lay upon a chair beside him. Altogether, look as I would, there was nothing remarkable about the man save his blazing red head, and the expression of extreme chagrin and discontent upon his features.Sherlock Holmes’ quick eye took in my occupation, and he shook his head with a smile as he noticed my questioning glances. “Beyond the obvious facts that he has at some time done manual labour, that he is a Freemason, that he has been in China, and that he has done a considerable amount of writing lately, I can deduce nothing else.”Mr. Jabez Wilson started up in his chair, with his fore?nger upon the paper, but his eyes upon my companion.“How, in the name of good-fortune, did you know all that, Mr. Holmes?” he asked. “How did you know, for example, that I did manual labour? It’s as true as gospel, for I began as a ship’s carpenter.”“Your hands, my dear sir. Your right hand is quite a size larger than your left. You have worked with it, and the muscles are more developed.”“Well, then, and the Freemasonry?”“I won’t insult your intelligence by telling you how I read that, especially as, rather against the strict rules of your order, you use an arc-and-compass breastpin.”“Ah, of course, I forgot that. But the writing?”“What else can be indicated by that right cuff so very shiny for ?ve inches, and the left one with the smooth patch near the elbow where you rest it upon the desk?”“Well, but China?”“The ?sh that you have tattooed immediately above your right wrist could only have been done in China. I have made a small study of tattoo marks and have even contributed to the literature of the subject. That trick of staining the ?shes’ scales of a delicate pink is quite peculiar to China. When, in addition, I see a Chinese coin hanging from your watch-chain, the matter becomes even more simple.”60Mr. Jabez Wilson laughed heavily. “Well, I never!” said he. “I thought at ?rst that you had done something clever, but I see that there was nothing in it after all.”“I begin to think, Watson,” said Holmes, “that I make a mistake in explaining. ‘Omne ignotum pro magni?co,’* you know, and my poor little reputation, such as it is, will suffer shipwreck if I am so candid. Can you not ?nd the advertisement, Mr. Wilson?”“Yes, I have got it now,” he answered with his thick red ?nger planted halfway down the column. “Here it is. This is what began it all. You just read it for yourself, sir.”*Roughly translated as “Everything unknown is/seems magni?cent.”QUESTION 1 OF 11Over the course of the passage, the main focus shifts from????????????????A an abstract description provided by the narrator to a concrete description provided by the client. B a detailed study of Sherlock Holmes to an outsider’s study of his client. C the description of a character’s outer appearance to a description of his inner beliefs. D the introduction of one character to the revelation of another character’s astute observations. QUESTION 2 OF 11As used in line 6 (“fashion”), “fashion” most nearly means????????????????A construction. B appearance. C manner. D clothing. QUESTION 3 OF 11The main purpose of the second paragraph is to????????????????A show that the client is less important than other clients who visit the of?ce. B prove that the narrator is inherently biased against the client. C demonstrate that the narrator is mistaken in his view of the world. D present the narrator’s assessment of the client based on his appearance. QUESTION 4 OF 11By the end of the passage, the client views Holmes’s deductions as????????????????A unimpressive. B intriguing. C astonishing. D insulting. QUESTION 5 OF 11The passage implies that Holmes knows that Watson????????A has been trying to make his own inferences about the client. B recognizes the client from a previous meeting. ????????????????????????????

??????????????????????????????????????????

????

????

C cannot understand why the client has chosen to come to them. D understands the client better than Holmes does.

??

??

D highlights the difference in social class between the detective and the client.

QUESTION 6 OF 11

Which choice provides the best evidence for the answer to the previous question?????

????

A lines 15–17(“A frayed top-hat and a faded brown overcoat with a wrinkled velvet collar lay upon a chair beside him.”)(“A frayed . . . him”) B lines 17–20(“Altogether, look as I would, there was nothing remarkable about the man save his blazing red

head, and the expression of extreme chagrin and discontent upon his features.”)(“Altogether . . . features”) C lines 21–23(“Sherlock Holmes’ quick eye took in my occupation, and he shook his head with a smile as he noticed my questioning glances.”)(“Sherlock . . . glances”) D lines 23–26(““Beyond the obvious facts that he has at some time done manual labour, that he is a Freemason, that he has been in China, and that he has done a

considerable amount of writing lately, I can deduce nothing else.””)(“Beyond . . . else”)

QUESTION 10 OF 11

It can be reasonably inferred that, after hearing Homes explain his observations, the client????????

????????

A will tell people that Holmes is a fraud. B no longer thinks that it is necessary to hire a detective for the case. C has a less romantic notion of Holmes’s abilities. D now considers Watson more intelligent than Holmes.

????

QUESTION 11 OF 11

Which choice provides the best evidence for the answer to the previous question???

??

A lines 52–54(“Mr. Jabez Wilson laughed heavily. “Well, I never!” said he. “I thought at ?rst that you had done

something clever, but I see that there was nothing in it after all.””)(“Mr. Jabez . . . all”) B lines 55–56(““I begin to think, Watson,” said Holmes, “that I make a mistake in explaining.”)(“I . . . explaining”) C lines 57–58(“my poor little reputation, such as it is, will suffer shipwreck if I am so candid.”)(“My poor . . . candid”) D lines 58–59(“Can you not ?nd the advertisement, Mr. Wilson?””)(“Can . . . Wilson”)

????

????

????

QUESTION 7 OF 11

In lines 46–48 (“I have made a small study of tattoo marks and have even contributed to the literature of the subject.”), the words “study,” and “literature” primarily serve to????????

????????

A demonstrate the depth of Holmes’s knowledge on an obscure topic. B provide a false sense of authority to Holmes’s deductions. C lure the client into giving more information than he intended to. D prove that Holmes feels that he is intellectually superior to Watson.

????

Find more here at AAA studio

QUESTION 8 OF 11

According to the passage, Holmes deduces that the client received his tattoo in China because????????

????????

A the client has said that he has spent time in China. B the tattoo is the Chinese symbol for the Freemasons. C Holmes wrote a book about the type of ?sh depicted in the tattoo. D the coloring technique used in the tattoo is unique to China.

QUESTION 9 OF 11

Sherlock Holmes’s deductions about the client serve to??????

??????

A introduce the client ironically to set the tone for the rest of the passage. B characterize Holmes as unusually perceptive and knowledgeable. C show the extent of Watson’s errors to establish him as incapable.

??????????????????????????????

??????????????????????????????????????????

ANSWER KEY

Exercise1Exercise2Exercise3Exercise4Exercise51234567891011BACDADCBDAA1234567891011ACACCCBDDCA1234567891011ACABDDCCBAC1234567891011BBACABDCDCB1234567891011DCDAACADBCAFind more here at AAA studio

??????????????????????????????

Reading Practices for the Redesigned SAT 1.2 Literature Level 4

Find more here at AAA studio

Proudly presented by Khan Academy Humbly brought to you by AAA Studio

??????????????????????????????????????????

Exercise 1

55

This passage is excerpted from Jack London, The Scarlet Plague. Originally published in 1915.

Line5

10

15

20

25

30

35

40

45

50

An old man and a boy travelled along this runway. They moved slowly, for the old man was very old, his movements tremulous, and he leaned heavily upon his staff. A rude skull-cap of goat-skin protected his head from the sun. From

beneath this fell a scant fringe of stained and dirty-white hair. A visor, ingeniously made from a large leaf, shielded his eyes, and from under this he peered at the way of his feet on the trail. His beard, which should have been snow-white but which showed the same weather-wear and camp-stain as his hair, fell nearly to his waist in a great tangled mass. About his chest and shoulders hung a single, mangy garment of goat-skin. His arms and legs, withered and skinny, betokened extreme age, as well as did their sunburn and scars and scratches betoken long years of exposure to the elements.The boy, who led the way, checking the eagerness of his muscles to the slow progress of the elder, likewise wore a single garment—a ragged-edged piece of bear-skin, with a hole in the middle through which he had thrust his head. He could not have been more than twelve years old. Tucked coquettishly over one ear was the freshly severed tail of a pig. In one hand he carried a medium-sized bow and an arrow.

On his back was a quiverful of arrows. From a sheath hanging about his neck on a thong, projected the battered handle of a hunting knife. He was as brown as a berry, and walked softly, with almost a catlike tread. In marked contrast with his sunburned skin were his eyes—blue, deep blue, but keen and sharp as a pair of gimlets. They seemed to bore into aft about him in a way that was habitual. As he went along he smelled things, as well, his distended, quivering nostrils carrying to his brain an endless series of messages from the outside world. Also, his hearing was acute, and had been so trained that it operated automatically. Without conscious effort, he heard all the slight sounds in the apparent quiet—heard, and differentiated, and classi?ed these sounds—whether they were of the wind rustling the leaves, of the humming of bees and gnats, of the distant rumble of the sea that drifted to him only in lulls, or of the gopher, just under his foot, shoving a pouchful of earth into the entrance of his hole.

Suddenly he became alertly tense. Sound, sight, and odor had given him a simultaneous warning. His hand went back to the old man, touching him, and the pair stood still. Ahead, at one side of the top of the embankment, arose a crackling sound, and the boy's gaze was ?xed on the tops of the agitated bushes. Then a large bear, a grizzly, crashed into view, and likewise stopped abruptly, at sight of the humans. He did not like them, and growled querulously. Slowly the boy ?tted the arrow to the bow, and slowly he pulled the bowstring taut. But he never removed his eyes from the bear.The old man peered from under his green leaf at the danger, and stood as quietly as the boy. For a few seconds this mutual

60

65

70

75

80

scrutinizing went on; then, the bear betraying a growing irritability, the boy, with a movement of his head, indicated that the old man must step aside from the trail and go down the embankment. The boy followed, going backward, still holding the bow taut and ready. They waited till a crashing among the bushes from the opposite side of the embankment told them the bear had gone on. The boy grinned as he led back to the trail.

\The old man shook his head.

%undependable falsetto. \the time when a man would be afraid of his life on the way to the Cliff House. When I was a boy, Edwin, men and women and little babies used to come out here from San Francisco by tens of thousands on a nice day. And there weren't any bears then. No, sir. They used to pay money to look at them in cages, they were that rare.\\

Before the old man could answer, the boy recollected and triumphantly shoved his hand into a pouch under his bear-skin and pulled forth a battered and tarnished silver dollar. The old man's eyes glistened, as he held the coin close to them.

\make out the date, Edwin.\The boy laughed.

\making believe them little marks mean something.”

QUESTION 1 OF 11

Over the course of the passage, the main focus shifts from????????

????????

A an appreciative characterization of the wilderness to a critique of civilization. B the depiction of a particular place and time to a broad prediction about the future. C a physical description of characters and their environment to a discussion of one character’s memories. D the opinions held by a young character to the views asserted by an older character.

QUESTION 2 OF 11

The narrator characterizes Edwin as someone who????????

????????

A has never known a different way of life. B does not respect his elders. C has just begun to develop survival skills. D refuses education in any form.

QUESTION 3 OF 11

According to the passage, Edwin’s sensory skills are??

??

A highly developed and give him a precise awareness of the environs.

????????????????????????????

??????????????????????????????????????????

??????

??????

B ?nely tuned but fail to protect Edwin from imminent dangers. C equally as pro?cient as Granser’s sensory abilities. D solely responsible for Edwin and Granser’s continued safety.

growled querulously.”) ??

??

C line 61 (“A . . . chuckled”)(“\chuckled.”) D lines 63–64 (“They . . . falsetto”)(“\day,\????

QUESTION 4 OF 11

In lines 8–10 (“His beard, which should have been snow-white but which showed the same weather-wear and camp-stain as his hair, fell nearly to his waist in a great tangled mass.”), the description of the beard mainly serves to????????

????????

A demonstrate that Granser is negligent. B emphasize Granser’s advanced age. C highlight Granser’s concern about appearances. D show that Granser intimidates others.

QUESTION 9 OF 11

It can reasonably be inferred from Edwin and Granser’s conversation that????????

????????

A citizens have recently become aware of changes in their environment. B the social changes Granser refers to are his own imaginary inventions. C society underwent a drastic change relatively early in Granser’s life. D Edwin and Granser have made a deliberate choice to leave society.

QUESTION 5 OF 11

In paragraph 3 (lines 26–32(“catlike tread. In marked contrast with his sunburned skin were his eyes—blue, deep blue, but keen and sharp as a pair of gimlets. They seemed to bore into aft about him in a way that was habitual. As he went along he smelled things, as well, his

distended, quivering nostrils carrying to his brain an endless series of messages from the outside world. Also, his hearing was acute,”)), the words “catlike,” “keen,” and “acute” primarily serve to????????

????????

A describe the way Edwin relates to animals. B characterize Edwin’s interaction with his surroundings. C describe Edwin’s innate personality traits. D reveal Edwin’s superiority to Granser.

QUESTION 10 OF 11

Which choice provides the best evidence for the answer to the previous question???

??

A lines 64–66(“\when a man would be afraid of his life on the way to the Cliff House.”)(“Who’d. . . House) B lines 66–68 (“When . . . day”)(“ When I was a boy, Edwin, men and women and little babies used to come out here from San Francisco by tens of thousands on a nice day. ”) C lines 72–74 (“Before . . . dollar”)(“Before the old man could answer, the boy recollected and triumphantly shoved his hand into a pouch under his bear-skin and pulled forth a battered and tarnished silver dollar.”) D line 77 (“I . . muttered”)(“\????

QUESTION 6 OF 11

As used in line 45 (“?xed on”), “?xed on” most nearly means????????

????????

A attached to. B locked on. C established on. D secured to.

????

????

QUESTION 11 OF 11

Edwin’s reaction to the date on the coin primarily serves to????????

????????

A demonstrate Edwin’s lack of intelligence. B represent a belief that is rapidly becoming common. C show that Edwin does not trust Granser. D emphasize the extent to which social practices have changed.

QUESTION 7 OF 11

Based on the description of the encounter with the bear, it can reasonably be inferred that????????

????????

A Granser and Edwin have previously come across bears on their journey. B Granser and Edwin are not concerned by the presence of animals in the forest. C Granser is surprised to see a bear that is not in a cage. D Edwin is distracted by the constant threat posed by wildlife.

QUESTION 8 OF 11

Which choice provides the best evidence for the answer to the previous question???

??

A lines 41–42 (“Sound . . . warning”)(“Sound, sight, and odor had given him a simultaneous warning.”) ??

??

B line 48 (“He . . . querulously”)(“He did not like them, and

????????????????????????????

??????????????????????????????????????????

Exercise 2

55

This passage is excerpted from J.D. Beresford, The Looking Glass. Originally published in 1921.

Line5

10

15

20

25

30

35

40

45

50

This was the ?rst communication that had come from her aunt in Rachel's lifetime.

\he passed the letter across the table.

Rachel looked ?rst at the signature. It seemed strange to see her own name there. It was as if her individuality, her very identity, was impugned by the fact that there should be two Rachel Deanes. Moreover there was a likeness between her aunt's autograph and her own, a characteristic turn in the looping of the letters, a hint of the same decisiveness and precision. If Rachel had been educated ?fty years earlier, she might have written her name in just that manner.

\still stared at the signature.

Rachel's eyelids drooped and her expression indicated a faint, suppressed intolerance of her father's remark. He said the same things so often, and in so precisely the same tone, that she had formed a habit of automatically rejecting the truth of certain of his statements. He had always appeared to her as senile. He had been over ?fty when she was born, and ever since she could remember she had doubted the

correctness of his information. She was, she had often told herself, \veneration for the more distant past, but none for her father's period. She had long since condemned alike the ethic and the aesthetic of the nineteenth century as represented by her father's opinions; so that, even now, when his familiar comment coincided so queerly with her own thought, she instinctively disbelieved him. Yet, as always, she was gentle in her answer. She condescended from the heights of her youth and vigour to pity him.

\Rachel was like, dear,\you've seen her?\

\\prided herself on being so modern. She read Darwin and

things like that. Altogether beyond me, I admit. Still, it seems to me that the old truths have endured, and will, in spite of all--in spite of all.\

Rachel straightened her shoulders and lifted her head; there was disdain in her face, but none in her voice as she replied: \

She was excited at the thought of meeting this traditional, almost mythical aunt whom she had so often heard about. Sometimes she had wondered if the personality of this remarkable relative had not been a ?gment of her father's imagination, long pondered, and reconstructed out of half-forgotten material. But this letter of hers that now lay on the breakfast table was admirable in character. There was

something of condescension and intolerance expressed in the very restraint of its tone. She had written a kindly letter, but

60

65

70

75

80

the kindliness had an air of pity. It was all consistent enough with what her father had told her.

Mr. Deane came out of his reminiscences with a sigh.

\you had better accept this invitation to stay with her. She is rich, almost wealthy; and I, as you know, have practically nothing to leave you—practically nothing. If she took a fancy to you...\

He sighed again, and Rachel knew that for the hundredth time he was regretting his own past weakness. He had been so foolish in money matters, frittering away his once considerable capital in aimless speculations.

\Rachel said. \the way, how old is she?\

\said, \three. Dear, dear. Fancy Rachel being seventy-three! I always think of her as being about your age. It seems so absurd to think of her as old….\

He continued his re?ections, but Rachel was not listening. He was asking for the understanding of the young; quite

unaware of his senility, reaching out over half a century to try to touch the comprehension and sympathy of his daughter. But she was already bent on her own adventure, looking forward eagerly to a visit to London that promised delights other than the inspection of the mysterious, traditional aunt whom she had so long known by report.

QUESTION 1 OF 11

Over the course of the passage, the main focus shifts from????????

????????

A a depiction of a family’s strained dynamic to a character’s wandering recollections of the distant past. B one character's reception of new information to a frank discussion of money and inheritances. C two characters' reactions to an unexpected message to a character's anticipation of a journey. D a comparison between two characters to one character’s thoughts about memories and aging.

QUESTION 2 OF 11

Which statement best describes Rachel's perspective regarding her aunt?????????

????????

A She is curious about her similarities to her aunt and intrigued by the idea of meeting her. B She is disdainful of her aunt because she has ignored her family for years. C She is suspicious of her aunt's intentions but excited about the possibility of an inheritance. D She is puzzled by her aunt's sudden interest in meeting her.

QUESTION 3 OF 11

In line 7 (“impugned”), “impugned” most nearly means

????????????????????????????

??????????????????????????????????????????

????????

????????

A contradicted. B challenged. C opposed. D resisted.

??????

??????

B detail the differences between Rachel's generation and her father's. C validate Rachel's sense that her father is untrustworthy. D provide insight into Rachel's view of the past.

QUESTION 4 OF 11

The narrator implies that Rachel’s attitude towards her father is????????

????????

A overtly disrespectful. B generally loving. C mildly contemptuous. D wholly patronizing.

QUESTION 8 OF 11

It can reasonably be inferred that the relationship between Rachel's father and her aunt????????

????????

A has been damaged by a misunderstanding. B is strained as a result of their past con?icts. C has become less tense as time has passed. D is uneasy due to their different ?nancial situations.

QUESTION 5 OF 11

Which choice provides the best evidence for the answer to the previous question???

??

A lines 15–16 (“Rachel’s . . . remark”)(“Rachel's eyelids drooped and her expression indicated a faint, suppressed intolerance of her father's remark.”) ??

??

B lines 22–23 (“She . . . modern”)(“She was, she had often told herself, \ C lines 23–25 (“She had . . . period”)(“She had a certain veneration for the more distant past, but none for her father's period.”) D lines 32–33 (“I should . . . said”)(“\almost have forgotten what Aunt Rachel was like, dear,\said.”) QUESTION 9 OF 11

Which choice provides the best evidence for the answer to the previous question???

??

A lines 13–14 (“You're . . . signature”)(“\in some ways,\signature.”) ??

??

B lines 33–36 (“How . . . disagreed”)(“\since you've seen her?\ C lines 61–64 (“He sighed . . . speculations”)(“He sighed again, and Rachel knew that for the hundredth time he was regretting his own past weakness. He had been so foolish in money matters, frittering away his once considerable capital in aimless speculations.”) D lines 68–72 (“There . . . old”)(“\months between us,\dear me, yes;—she must be seventy-three. Dear, dear. Fancy Rachel being seventy-three! I always think of her as being about your age. It seems so absurd to think of her as old….\????

????

????

????

QUESTION 6 OF 11

As used in line 24 (“veneration”), “veneration” most nearly means????????

????????

A distaste. B reverence. C deference. D awe.

QUESTION 10 OF 11

Which conclusion does Rachel make based on the letter from her aunt?????????

????????

A Her aunt’s personality is consistent with her father’s stories. B Her father has made up a great deal about her due to his forgetfulness. C Her aunt has taken pity on them and wants to leave them her fortune. D She and her aunt are remarkably alike in personality.

QUESTION 7 OF 11

The primary purpose of paragraph 5 (lines 15–31 (“Rachel's eyelids drooped and her expression indicated a faint, suppressed intolerance of her father's remark. He said the same things so often, and in so

precisely the same tone, that she had formed a habit of automatically rejecting the truth of certain of his statements. He had always appeared to her as senile. He had been over ?fty when she was born, and ever since she could remember she had doubted the correctness of his information. She was, she had often told herself, %ultra-modern.\but none for her father's period. She had long since condemned alike the ethic and the aesthetic of the nineteenth century as represented by her father's opinions; so that, even now, when his familiar comment coincided so queerly with her own thought, she instinctively disbelieved him. Yet, as always, she was gentle in her answer. She condescended from the heights of her youth and vigour to pity him.”)) is to??

??

A establish Rachel's lack of interest in ethics and aesthetics.

QUESTION 11 OF 11

????????????????????????????

??????????????????????????????????????????

In lines 56–60 (“\think you had better accept this invitation to stay with her. She is rich, almost wealthy; and I, as you know, have practically nothing to leave you—practically nothing. If she took a fancy to you...\about wealth serves mainly to????????

????????

A emphasize that Rachel’s aunt makes strong demands of her family. B provide context for the disagreement between Rachel’s aunt and her father. C illustrate why Rachel generally disregards her father’s advice. D reveal that Rachel may bene?t from building a relationship with her aunt.

Find more here at

AAA studio

????????????????????????????

??????????????????????????????????????????

Exercise 3

55

This passage is excerpted from Katherine Mans?eld, The Voyage. Originally published in 1921.

Line5

10

15

20

25

30

35

40

45

50

The Picton boat was due to leave at half-past eleven. It was a beautiful night, mild, starry, only when they got out of the cab and started to walk down the Old Wharf that jutted out into the harbour, a faint wind blowing off the water ruf?ed under Fenella's hat, and she put up her hand to keep it on. It was dark on the Old Wharf, very dark; the wool sheds, the cattle trucks, the cranes standing up so high, the little squat railway engine, all seemed carved out of solid darkness. Here and there on a rounded wood-pile, that was like the stalk of a huge black mushroom, there hung a lantern, but it seemed afraid to unfurl its timid, quivering light in all that blackness; it burned softly, as if for itself.

Fenella's father pushed on with quick, nervous strides.

Beside him her grandma bustled along in her crackling black ulster; they went so fast that she had now and again to give an undigni?ed little skip to keep up with them. As well as her luggage strapped into a neat sausage, Fenella carried clasped to her her grandma's umbrella, and the handle, which was a swan's head, kept giving her shoulder a sharp little peck as if it too wanted her to hurry. Men, their caps pulled down, their collars turned up, swung by; a few women all muf?ed scurried along; and one tiny boy, only his little black arms and legs showing out of a white woolly shawl, was jerked along angrily between his father and mother; he looked like a baby ?y that had fallen into the cream.

Then suddenly, so suddenly that Fenella and her grandma both leapt, there sounded from behind the largest wool shed, that had a trail of smoke hanging over it, \\they came in sight of the Picton boat. Lying beside the dark wharf, all strung, all beaded with round golden lights, the Picton boat looked as if she was more ready to sail among stars than out into the cold sea. People pressed along the gangway. First went her grandma, then her father, then

Fenella. They stepped out of the way of the hurrying people, and standing under a little iron stairway that led to the upper deck they began to say goodbye.

\giving grandma another strapped-up sausage.\

\\

\

Grandma felt for them inside her glove and showed him the tips.

\

He sounded stern, but Fenella, eagerly watching him, saw that he looked tired and sad.

\heads, and a voice like a cry shouted, \gangway?\

60

65

70

75

\say. And her grandma, very agitated, answered, \will, dear. Go now. You'll be left. Go now, Frank. Go now.\\surprise Fenella saw her father take off his hat. He clasped grandma in his arms and pressed her to him. \mother!\

And grandma put her hand, with the black thread glove that was worn through on her ring ?nger, against his cheek, and she sobbed, \

This was so awful that Fenella quickly turned her back on them, swallowed once, twice, and frowned terribly at a little green star on a mast head. But she had to turn round again; her father was going.

\

moustache brushed her cheek. But Fenella caught hold of the lapels of his coat.

\He wouldn't look at her. He shook her off gently, and gently said, \pressed something into her palm. \you should need it.\

A shilling! She must be going away for ever! \Fenella. But he was gone. He was the last off the ship. The sailors put their shoulders to the gangway. A huge coil of dark rope went ?ying through the air and fell \wharf. A bell rang; a whistle shrilled. Silently the dark wharf began to slip, to slide, to edge away from them. Now there was a rush of water between. Fenella strained to see with all her might. Was that father turning round? Or waving? Or standing alone? Or walking off by himself? The strip of water grew broader, darker. Now the Picton boat began to swing round steady, pointing out to sea. It was no good looking any longer.

QUESTION 1 OF 11

Which choice best summarizes the passage?????????

????????

A A girl and her family brace for a long voyage overseas and prepare for a new chapter in their lives. B A girl and her family exchange goodbyes and experience an emotional departure. C A girl and her family adjust to a new living situation and resign themselves to an unclear future. D A girl and her family re?ect on their changing relationships and settle into their new roles.

QUESTION 2 OF 11

Over the course of the passage, the main focus shifts from??????

??????

A a ship’s nighttime departure to the devastating emotional impact of a family’s separation. B a family’s protracted farewell to a description of a ship’s sailors preparing to depart. C a description of a boat dock to an exchange of money between family members.

????????????????????????????

??????????????????????????????????????????

??

??

D a vivid illustration of a setting to the sense of resignation one character experiences.

You'll be left. Go now, Frank. Go now.\??

??

C lines 56–57 (“It’s . . . hat”)(“\another three minutes.\father take off his hat.”) D lines 60–62 (“And . . .son”)(“And grandma put her hand, with the black thread glove that was worn through on her ring ?nger, against his cheek, and she sobbed, \you, my own brave son!\QUESTION 3 OF 11

The author includes a lengthy description of the Old Wharf most likely to????????

????????

A show that the harbor is an important setting that emphasizes the family’s separation. B provide a vivid visual backdrop that underscores the importance of what is left behind. C establish a tense and hurried atmosphere that foreshadows the action of the story. D emphasize how the wharf at night creates an unsettling ambiance for the passers-by.

????

QUESTION 8 OF 11

As used in line 63 (“awful”), \????????

????????

A distressing. B disgusting. C shocking. D appalling.

QUESTION 4 OF 11

What main effects do the words “timid” and “quivering,” used in the ?rst paragraph, have on the tone of the passage?????????

????????

A They create an ominous tone that foreshadows Fenella’s separation from her father. B They create a solemn tone that contrasts with Fenella’s emotions about leaving. C They create an uncertain tone that re?ects Fenella’s relationship with her father. D They create a sinister tone that implies Fenella’s wariness of the situation.

QUESTION 9 OF 11

Which choice best describes Fenella's attitude toward her imminent journey?????????

????????

A She is de?ant in the face of an unclear future. B She is worried about her impending separation from her father. C She is frightened about the journey by boat she is about to take. D She is resolute in her belief that she will be reunited with her father soon.

QUESTION 5 OF 11

It can be reasonably inferred that Fenella’s father “looked tired and sad” (line 48 (“looked tired and sad.”)) because he????????

????????

A has a long journey ahead of him. B feels concerned about his family’s tickets. C is going to miss his family. D senses something is wrong.

QUESTION 10 OF 11

Which choice provides the best evidence for the answer to the previous question???

??

A lines 68–71 (“But . . . her”)(“But Fenella caught hold of the lapels of his coat.”) ??

??

B lines 75–76 (“Father . . . ship”)(“\But he was gone. He was the last off the ship.”) C lines 81–83 (“Fenella . . . himself”)(“Fenella strained to see with all her might. Was that father turning round? Or waving? Or standing alone? Or walking off by himself?”) D lines 84–86 (“Now . . . longer”)(“Now the Picton boat began to swing round steady, pointing out to sea. It was no good looking any longer.”) QUESTION 6 OF 11

Which statement best characterizes the relationship between Fenella’s father and grandmother?????????

????????

A Their relationship is complicated by quiet disagreements. B Their relationship is loving and includes tender sentiments. C Their relationship is troubled because of tense animosity. D Their relationship is restrained and based on polite affection.

????

????

QUESTION 7 OF 11

Which choice provides the best evidence for the answer to the previous question???

??

A lines 38–40 (“There . . . Frank”)(“\your luggage!\another strapped-up sausage.”) ??

??

B lines 52–55 (“You’ll . . . now”)(“\father,\very agitated, answered, \

QUESTION 11 OF 11

At the end of the passage, Fenella is convinced that she will be away for a very long time because??????

??????

A her father has given her what she considers to be a great deal of money. B her father has refused to answer her about the length of her travels. C her father has said goodbye and told her to “be a good girl.”

????????????????????????????

??????????????????????????????????????????

??

??

D her father has exchanged an emotional farewell with her grandmother.

Find more here at

AAA studio

????????????????????????????

??????????????????????????????????????????

Exercise 4

55

This passage is excerpted from Edith Wharton, House of Mirth, originally published in 1905.

Line5

10

15

20

25

30

35

40

45

50

Selden paused in surprise. In the afternoon rush of the Grand Central Station his eyes had been refreshed by the sight of Miss Lily Bart.

It was a Monday in early September, and he was returning to his work from a hurried dip into the country; but what was Miss Bart doing in town at that season? Her desultory air perplexed him. She stood apart from the crowd, letting it drift by her to the platform or the street, and wearing an air of irresolution which might, as he surmised, be the mask of a very de?nite purpose. It struck him at once that she was waiting for someone, but he hardly knew why the idea

arrested him. There was nothing new about Lily Bart, yet he could never see her without a faint movement of interest: it was characteristic of her that she always roused speculation, that her simplest acts seemed the result of far-reaching intentions.

An impulse of curiosity made him turn out of his direct line to the door, and stroll past her. He knew that if she did not wish to be seen she would contrive to elude him; and it amused him to think of putting her skill to the test. \Selden—what good luck!\

She came forward smiling, eager almost, in her resolve to intercept him. One or two persons, in brushing past them, lingered to look; for Miss Bart was a ?gure to arrest even the suburban traveller rushing to his last train.

Selden had never seen her more radiant. Her vivid head, relieved against the dull tints of the crowd, made her more conspicuous than in a ball-room, and under her dark hat and veil she regained the girlish smoothness, the purity of tint, that she was beginning to lose after eleven years of late hours and indefatigable dancing.

\rescue!\

He responded joyfully that to do so was his mission in life, and asked what form the rescue was to take.

\me. One sits out a cotillion—why not sit out a train? It isn't a bit hotter here than in Mrs. Van Osburgh's conservatory—and some of the women are not a bit uglier.\laughing, to explain that she had come up to town from Tuxedo, on her way to the Gus Trenors' at Bellomont, and had missed the three-?fteen train to Rhinebeck. \isn't another till half-past ?ve.\

jewelled watch among her laces. \I don't know what to do with myself. My maid came up this morning to do some shopping for me, and was to go on to Bellomont at one o'clock, and my aunt's house is closed, and I don't know a soul in town.\the station. \you can spare the time, do take me somewhere for a breath of air.\

60

65

He declared himself entirely at her disposal: the adventure struck him as diverting. As a spectator, he had always

enjoyed Lily Bart; and his course lay so far out of her orbit that it amused him to be drawn for a moment into the sudden intimacy which her proposal implied.

\

She smiled assentingly, and then made a slight grimace.\sure to meet a lot of bores. I'm as old as the hills, of course, and it ought not to make any difference; but if I'M old

enough, you're not,\but isn't there a quieter place?\

He answered her smile, which rested on him vividly. Her discretions interested him almost as much as her

imprudences: he was so sure that both were part of the same carefully-elaborated plan. In judging Miss Bart, he had always made use of the \

QUESTION 1 OF 11

Which choice best summarizes the passage?????????

????????

A The passage presents a portrait of two characters who decide to travel together B The passage explains the reasons for one character’s avoidance of a community C The passage captures one character’s fascination with another character D The passage describes a busy train station and the characters who inhabit it

QUESTION 2 OF 11

Over the course of the passage, Selden’s attitude shifts from????????

????????

A bewildered to curious. B dismayed to apprehensive. C amazed to anticipatory. D incredulous to indifferent.

QUESTION 3 OF 11

Which statement best characterizes the relationship between Selden and Lily?????????

????????

A Selden is skeptical of Lily and thinks she is hiding something from him B Selden is captivated by Lily and wants to spend more time with her C Selden is alarmed by Lily’s behavior and strains to understand her D Selden is smitten with Lily and begins to fall in love with her

QUESTION 4 OF 11

In the passage, Lily is characterized as someone who is??

??

A intriguing and deliberate.

????????????????????????????

??????????????????????????????????????????

??????

??????

B irritable and anxious. C surprising and irreverent. D secretive and moody.

??????

??????

B is worried she won’t make it to her destination on time. C is sure that it will be too hot and crowded to enjoy. D believes she is too old to enjoy the atmosphere.

QUESTION 5 OF 11

Which choice provides the best evidence for the answer to the previous question???

??

A lines 7–10 (“She . . . purpose”)(“She stood apart from the crowd, letting it drift by her to the platform or the street, and wearing an air of irresolution which might, as he surmised, be the mask of a very de?nite purpose.”) ??

??

B lines 10–12 (“It . . . him”)(“It struck him at once that she was waiting for someone, but he hardly knew why the idea arrested him.”) C lines 12–16 (“There . . . intentions”)(“There was nothing new about Lily Bart, yet he could never see her without a faint movement of interest: it was characteristic of her that she always roused speculation, that her simplest acts seemed the result of far-reaching intentions.”) D lines 18–20(“He knew that if she did not wish to be seen she would contrive to elude him; and it amused him to think of putting her skill to the test.”)(He . . . test”)

QUESTION 10 OF 11

Selden’s observations of Lily primarily indicate that he????????

????????

A is cautiously wary of Lily’s actions. B is pleased to be in Lily's company. C thinks that Lily can be a manipulative person. D feels drawn to Lily in spite of himself.

????

QUESTION 11 OF 11

Which choice provides the best evidence for the answer to the previous question???

??

A lines 52–53 (“He . . . diverting”)(“He declared himself entirely at her disposal: the adventure struck him as diverting.”) ??

??

B lines 54–56 (“and . . . implied”)(“and his course lay so far out of her orbit that it amused him to be drawn for a moment into the sudden intimacy which her proposal implied.”) C line 58 (“She . . . grimace”)(“She smiled assentingly, and then made a slight grimace.”) D line 64 (“He . . . vividly”)

????

????

QUESTION 6 OF 11

As used in line 23 (“intercept”), “intercept” most nearly means????????

????????

A halt. B de?ect. C overhear. D possess.

????

Find more here at AAA studio

QUESTION 7 OF 11

As used in line 26 (“vivid”), “vivid” most nearly means????????

????????

A bright. B animated. C clear. D sharp.

QUESTION 8 OF 11

The conversation between Selden and Lily in lines 32–39 (“What . . . uglier”) (“\rescue!””) serves primarily to????????

????????

A show how long it has been since they last saw each other. B demonstrate their shared experiences. C introduce them as potential romantic partners. D establish the imbalance of power between them.

QUESTION 9 OF 11

It can be inferred that Lily does not want to go to Sherry’s for tea because she??

??

A thinks she will not ?nd the company there engaging.

??????????????????????????????

??????????????????????????????????????????

ANSWER KEY

Exercise1Exercise2Exercise3Exercise41234567891011CAABBBADCAD1234567891011CABCABDBBAD1234567891011BDCACBDABAA1234567891011CABACAABABBFind more here at AAA studio

??????????????????????????????